Insight Ias Prelims 2019 Test 10 Solutions.pdf

  • Uploaded by: Pruthvi Nath
  • 0
  • 0
  • August 2019
  • PDF

This document was uploaded by user and they confirmed that they have the permission to share it. If you are author or own the copyright of this book, please report to us by using this DMCA report form. Report DMCA


Overview

Download & View Insight Ias Prelims 2019 Test 10 Solutions.pdf as PDF for free.

More details

  • Words: 23,210
  • Pages: 84
Mock Test 10 https://telegram.me/UPSCMaterials

https://telegram.me/FreeUPSCMaterials

https://telegram.me/MaterialforExam

1 With reference to the Ministry of Defence’s (MoD) prestigious Strategic Partnership (SP) Model, consider the following statements. 1. The model seeks to identify Indian private companies as Strategic Partners who would tie up with a few shortlisted foreign Original Equipment Manufacturers (OEMs) to manufacture important military platforms in India. 2. Any applicant company interested in participating in the selection process for strategic partners must be owned and controlled by resident Indians. 3. Existing Strategic Partners would have the advantage of being the automatic choice for the relevant future contracts of the Ministry of Defence. Select the correct answer using the codes below. A. 1 only B. 2 and 3 only C. 2 only D. 1 and 2 only orrect Answer : D

Answer Justification : Context: The Defence Acquisition Council (DAC), in a landmark decision, has approved procurement of 111 Utility Helicopters for the Indian Navy at a cost of over Rs. 21,000 crores. This is the first project under the With reference to the Ministry of Defence’s (MoD) prestigious Strategic Partnership (SP) Model that aims at providing significant fillip to the Government’s ‘Make in India’ programme. Concept: The strategic partner model is intended to enhance competition, increase efficiencies, facilitate faster and more significant absorption of technology, create a tiered industrial ecosystem, ensure development of a wider skill base, trigger innovation and enable participation in global value chains as well as promote exports. Under the model, the government intends to boost private sector participation and create domestic expertise in four key areas, namely, fighter aircraft, helicopters, submarines, and armoured vehicles and main battle tanks. One company would be selected for each area based on its competence, which would then tie up with the foreign Original Equipment Manufacturer selected through the procurement process, to build the platform in India with significant technology transfer. The model, whose concept was first suggested by the Dhirendra Singh Committee in its July 2015 report, populates Chapter VII of the Defence Procurement Procedure 2016 (DPP 2016). The ultimate aim of the model is to enhance India’s self-reliance index in defence procurement which continues to remain at an abysmally low level despite a huge defence industrial complex much of which is managed by state-owned Defence Public Sector Undertakings (DPSUs) and the Ordnance Factory Board (OFB).

1

Mock Test 10 https://telegram.me/UPSCMaterials

https://telegram.me/FreeUPSCMaterials

https://telegram.me/MaterialforExam

Justification: Statement 1: The strategic partnership model seeks to identify a few Indian private companies as Strategic Partners who would initially tie up with a few shortlisted foreign Original Equipment Manufacturers (OEMs) to manufacture big-ticket military platforms. In the initial phase, the selection of SPs would be confined to four segments: Fighter Aircraft, Helicopters, Submarines, and Armoured Fighting Vehicles (AFV)/Main Battle Tanks (MBT). In each segment, “only one SP would generally be selected”. Statement 2: According to the guidelines stipulated in the new Chapter VII of DPP 2016, any applicant company interested in participating in the selection process for strategic partners must be owned and controlled by resident Indians. This means that the majority in a company’s board of directors, including the Chief Executive Officer (CEO), must be resident Indians, and that a minimum 51 per cent of its equity must be owned by resident Indians. The cap of a maximum of 49 per cent Foreign Direct Investment (FDI) in SPs, which is also the condition in the newly revised ‘Make’ procedure, is intended to keep the crucial decision-making and intellectual property rights (IPR) in the hands of resident Indians. Statement 3: It is to be noted, however, that existing Strategic Partners would not be the automatic choice for future contracts, although they would be given some weightage in the tendering process for the core expertise developed during the execution of the initial contract under the Strategic Partnership model. Q Source: http://www.insightsonindia.com/2018/08/27/insights-daily-current-affairs-27-august-2018/ https://idsa.in/idsacomments/strategic-partnership-model-in-defence-industry_lkbehera_020617 2 Consider the distribution of water on earth. 1. Ice caps hold more water than salt lakes and atmosphere combined. 2. Groundwater is a richer source of water than both fresh water lakes and inland seas. 3. Rivers hold more water than lakes. Select the correct answer using the codes below. A. 1 and 2 only B. 2 and 3 only C. 1 only D. 1 and 3 only

Correct Answer : A Answer Justification : Justification:

2

Mock Test 10 https://telegram.me/UPSCMaterials

https://telegram.me/FreeUPSCMaterials

https://telegram.me/MaterialforExam

Q Source: Chapter 5: 7th NCERT Geography 3 HOPE Probe project, concerned with discoveries on Mars, has been initiated by A. B. C. D.

Russia USA United Arab Emirates China

Correct Answer : C Answer Justification : Learning: The probe will be built by an Emirati team of engineers and experts and will be sent on a scientific voyage of discovery to the Red Planet. This will mark the Arab world’s entry into the era of space exploration and place the UAE among the major scientific countries that have begun programmes to explore Mars. The probe will be sent to explore the Red Planet by 2020. Following a journey of several months, the probe is expected to enter the Red Planet’s orbit in 2021, coinciding with the 50th anniversary of the formation of the UAE. Scientific Objectives of the Probe Voyage: The Emirates Mars Mission project will answer scientific questions that have long puzzled scientists. These are questions about the Red Planet, which scientists have not been able to explain before because of the lack of data and information. The project will cover all aspects that have not been previously covered, whether scientific or knowledge-based, and it will work on drawing a clear and comprehensive picture of the 3

Mock Test 10 https://telegram.me/UPSCMaterials

https://telegram.me/FreeUPSCMaterials

https://telegram.me/MaterialforExam

Martian climate and the causes of the corrosion of its surface that has made it impossible for water to exist on the planet. The project will also provide insights about the weather on the Red Planet. It will observe weather phenomena such as dust storms and changes in temperature and how the atmosphere interacts with topography, from the highest volcano peaks to ice sheets to the vast deserts and the deepest canyons. Q Source: http://www.insightsonindia.com/2018/09/04/insights-daily-current-affairs-04-september-2018/ 4 Consider the following about superbugs. 1. A superbug is resistant to all anti-biotics. 2. Most superbugs are gram positive bacteria. Which of the above is/are correct? A. 1 only B. 2 only C. Both 1 and 2 D. None

Correct Answer : D Answer Justification : Justification: Statement 1: A superbug, also called multiresistant, is a bacterium that carries several resistance genes. These are resistant to multiple antibiotics and are able to survive even after exposure to one or more antibiotics. A superbug is usually defined as a microorganism that’s resistant to commonly used antibiotics – but not all superbugs are created equal. The number of different antibiotics to which it can be resistant determines the degree of the superbug. Some are resistant to one or two, but others can be resistant to multiple drugs. So, if a bug is resistant to every available antibiotic, it would be the superbug of all superbugs. Statement 2: There are two major classes of bacteria, known as Gram positive and Gram negative. They take their names from how they respond to the Gram staining test, which in turn was named after Danish scientist Hans Christian Gram, who developed the technique. A bacterium is known to be Gram negative or Gram positive based on its reaction to the test – Gram positive bacteria stain purple, and Gram negative do not. Gram negative bacteria are generally considered the more difficult to treat. They include such nasties as E. coli, Salmonella, Pseudomonas, and the Gonococcus bacteria – responsible for the 4

Mock Test 10 https://telegram.me/UPSCMaterials

https://telegram.me/FreeUPSCMaterials

https://telegram.me/MaterialforExam

sexually transmitted infection gonorrhoea. Antibiotics have a tough time dealing with Gram negative bacteria because of their additional outer membrane, which prevents drugs from getting inside. And if the drugs do manage to get in there, the bacteria have a pumping mechanism that forces them back out again so quickly, they don’t have time to work. Australian scientists have warned that Staphylococcus epidermidis, a superbug resistant to all known antibiotics that can cause “severe” infections or even death is spreading undetected through hospital wards across the world. Learning: Researchers discovered three variants of the multidrug-resistant bug in samples from 10 countries, including strains in Europe that cannot be reliably tamed by any drug currently on the market. The bacteria, known as Staphylococcus epidermidis, is related to the better-known and more deadly MRSA superbug. It’s found naturally on human skin and most commonly infects the elderly or patients who have had prosthetic materials implanted, such as catheters and joint replacements. Q Source: http://www.insightsonindia.com/2018/09/04/insights-daily-current-affairs-04-september-2018/ 5 Ocean waves do NOT transfer X from one place to another. 1. Wave energy 2. Phytoplankton floating on Ocean water Which of the above is/are correct? A. 1 only B. 2 only C. Both 1 and 2 D. None

Correct Answer : B Answer Justification : Justification: A is a disturbance that transfers energy from one place to another. Waves can transfer energy over distance without moving matter the entire distance. For example, an ocean wave can travel many kilometers without the water itself moving many kilometers. The water moves up and down—a motion known as a disturbance. It is the disturbance that travels in a wave, transferring energy. Learning: Waves are most commonly caused by wind. Wind-driven waves, or surface waves, are created by the friction between wind and surface water. As wind blows across the surface of the 5

Mock Test 10 https://telegram.me/UPSCMaterials

https://telegram.me/FreeUPSCMaterials

https://telegram.me/MaterialforExam

ocean or a lake, the continual disturbance creates a wave crest. These types of waves are found globally across the open ocean and along the coast. The gravitational pull of the sun and moon on the earth also causes waves. These waves are tides or, in other words, tidal waves. It is a common misconception that a tidal wave is also a tsunami. The cause of tsunamis is not related to tide information at all but can occur in any tidal state. th

Q Source: Additional Research on Chapter 5: 7 NCERT Geography 6 The practice of “Fiscal Federalism” in India involves A. B. C. D.

Distribution of financial powers between Centre and States Setting up of Finance Commission every five years Devolution of central pool to states All of the above

Correct Answer : D Answer Justification : Learning: Federalism refers to the division of responsibilities and functions between the Centre and States. Fiscal federalism is an important component of federalism. It refers to all financial matters conducted between the Centre and states with a view of a political federal structure in mind. For e.g. the 14th FC has radically enhanced the share of the states in the central divisible pool from the current 32 percent to 42 per cent which is the biggest ever increase in vertical tax devolution. This is supposed to strengthen fiscal federalism. Q Source: General terms used in India Polity: Often in news 7 Why Higher Education Institutions (HEIs) would want to join the Innovation Cell (ICs) Network in India? 1. Students and Faculty associated with ICs will have exclusive opportunity to participate in various Innovation related initiatives organized by MHRD. 2. No separate major capital investment is required to join the IC and the IC will make use of existing local ecosystem in the HEIs to promote innovation. Which of the above is/are correct? A. 1 only B. 2 only C. Both 1 and 2 D. None

Correct Answer : C 6

Mock Test 10 https://telegram.me/UPSCMaterials

https://telegram.me/FreeUPSCMaterials

https://telegram.me/MaterialforExam

Answer Justification : Justification: Both statements are correct. Innovation cell is MHRD’s initiative established at AICTE with a purpose to systematically foster the culture of Innovation in all Higher Education Institutions (HEIs) across the country. The primary mandate of Innovation Cell is to encourage, inspire and nurture young students by exposing them to new ideas and processes resulting in innovative activities in their formative years fostered through Network of Innovation clubs in Higher Educational Institutions. Major Programs Network of Innovation Clubs (NIC) Atal Ranking of Institutions on Innovation Achievements (ARIIA) Smart India Hackathon (SIH) 2019 National Student Startup Policy (NSSP) Network of Innovation Clubs (NIC) MIC will prescribe basic framework for the structure, formation, structure and smooth running of ICs and will soon share list of different activities which ICs need to undertake for the entire academic year. Q Source: http://pib.nic.in/newsite/PrintRelease.aspx?relid=183177 8 Equatorial counter-currents are unique because A. B. C. D.

They flow in a direction opposite to that of the surface winds They circulate from equator the poles uninterrupted. Their travel speed is not affected by the ocean depth. They are the only current to be sandwiched between two eastward-flowing ocean currents.

Correct Answer : A Answer Justification : Justification & Learning: This will address the concept comprehensively. Equatorial counter-currents are major surface flows that carry water eastward in the Atlantic, Indian, and Pacific Oceans. 7

Mock Test 10 https://telegram.me/UPSCMaterials

https://telegram.me/FreeUPSCMaterials

https://telegram.me/MaterialforExam

They are located near the equator and are sandwiched between two westward-flowing currents, the North Equatorial Current and the South Equatorial Current. So, option D is incorrect. Equatorial counter-currents are unique, in that they flow in the opposite direction of the surface winds. The other major surface currents in the tropics flow in the same direction as the prevailing winds. The equatorial counter-currents are thus driven by a distinct surface wind pattern in the tropics. Strong westward trade winds result in westward surface flow in most of the tropical Atlantic and Pacific Oceans. However, several hundred miles north of the equator the winds are much weaker, in comparison. The stronger winds to the south pile up water where the winds are weak. As a result, the surface of the ocean can be up to 6 inches higher. The excess water flows eastward under the influence of the Earth’s rotation, giving rise to the equatorial countercurrents.

Q Source: Improvisation: Fig 14.3: Page 124: Fundamentals of Physical Geography: 11th NCERT 9 Which of the following is NOT vital for the establishment or success of a democracy? A. Presence of Rule of Law B. Granting fundamental rights to the population 8

Mock Test 10 https://telegram.me/UPSCMaterials

https://telegram.me/FreeUPSCMaterials

https://telegram.me/MaterialforExam

C. Political equality of citizens to run for public offices D. Direct elections of all representatives

Correct Answer : D Answer Justification : Justification: Option A: Rule of law ensures that democracy doesn’t turn into Rule of Men. Option B: Fundamental rights guard the citizens from the tyranny and despotism of the rulers. Option C: If a certain class of citizens are banned (without any intelligible criteria) from participating in elections or running for public offices, a democracy may not be sustained. Option D: Many democracies practice indirect election of representatives, for e.g. President of USA is indirectly elected. Voters elect the Electoral College, which then elects the President. In India, Rajya Sabha MPs are indirectly elected. So, direct election of all representatives is not crucial for a democracy. Q Source: Revision: Concepts covered in previous tests 10 Aeolus, or, Atmospheric Dynamics Mission Aeolus, is an Earth observation satellite. Consider the following about it. 1. It has been built by European Space Agency (ESA). 2. It will be the first satellite with equipment capable of performing global wind-component-profile observation from space. Which of the above is/are correct? A. 1 only B. 2 only C. Both 1 and 2 D. None

Correct Answer : B Answer Justification : Learning: ADM-Aeolus will be the first satellite with equipment capable of performing global windcomponent-profile observation and will provide much-needed information to improve weather forecasting. Aeolus will be the first satellite capable of observing what the winds are doing on Earth, from the surface of the planet and into the stratosphere 30 km high. The Aeolus is the fifth planned satellite in the Living Planet Programme of the European Space 9

Mock Test 10 https://telegram.me/UPSCMaterials

https://telegram.me/FreeUPSCMaterials

https://telegram.me/MaterialforExam

Agency (ESA). The main goal of this mission is to further develop the knowledge of Earth's atmosphere and weather systems. By recording and monitoring the weather in different parts of the world, Aeolus will allow scientists to build complex weather models, which can then be used to help predict how that environment will behave in the future. These predictions will be useful in the short-term, since they can be applied to Numerical Weather Prediction in order to make forecasts more accurate. The mission will thus improve the knowledge of all sorts of weather phenomena, from global warming to the effects of air pollution. Aeolus is seen as a mission that will pave the way for future operational meteorological satellites dedicated to study Earth's wind profiles.

Q Source: http://www.esa.int/Our_Activities/Operations/Aeolus_operations 10

Mock Test 10 https://telegram.me/UPSCMaterials

https://telegram.me/FreeUPSCMaterials

11 Consider the following. Communities seen in news 1. Tuaregs 2. Sinhalas 3. Rakhine

https://telegram.me/MaterialforExam

Belong to A. Sahara Desert B. Bhutan C. Sri Lanka

Which of these is/are correctly matched? A. 2 and 3 B. 1 only C. 1 and 3 only D. 1 and 2 only

Correct Answer : B Answer Justification : Justification: Statement 1: These are people with a nomadic pastoralist lifestyle. They are the principal inhabitants of the vast Sahara Desert. They raged the Tuareg Rebellion (2012) which was an early stage of the Northern Mali conflict waged against the Malian government. Their goal was attaining independence for the northern region of Mali, known as Azawad. Statement 2: Sinhalas are the major community of Sri Lanka often in conflict with the Tamils of SL. Statement 3: Rakhine is a state in Myanmar, and also a principal ethnic community. The 2012 Rakhine State riots were a series of conflicts primarily between ethnic Rakhine Buddhists and Rohingya Muslims in northern Rakhine State. th

Q Source: Often in news + Additional Research: Page 73: Geography NCERT 7 : Our Environment 12 With reference to Open-source software (OSS), consider the following statements. 1. It is a type of computer software whose source code is released without any license. 2. Open-source software may be developed in a collaborative public manner. Which of the above is/are correct? A. 1 only B. 2 only C. Both 1 and 2 D. None orrect Answer : B

Answer Justification : 11

Mock Test 10 https://telegram.me/UPSCMaterials

https://telegram.me/FreeUPSCMaterials

https://telegram.me/MaterialforExam

Justification: Statement 1: OSS is released under a license in which the copyright holder grants users the rights to study, change, and distribute the software to anyone and for any purpose. Open-source software may be developed in a collaborative public manner. According to scientists who studied it, open-source software is a prominent example of open collaboration. The term is often written without a hyphen as "open source software" The Open Source Definition, notably, presents an open-source philosophy, and further defines the terms of use, modification and redistribution of open-source software. Software licenses grant rights to users which would otherwise be reserved by copyright law to the copyright holder. Several open-source software licenses have qualified within the boundaries of the Open Source Definition. The most prominent and popular example is the GNU General Public License (GPL), which "allows free distribution under the condition that further developments and applications are put under the same licence", thus also free The Open Source Initiative's (OSI) definition is recognized by several governments internationally as the standard or de facto definition. In addition, many of the world's largest open-source-software projects and contributors, including Debian, Drupal Association, FreeBSD Foundation, Linux Foundation, Mozilla Foundation, Wikimedia Foundation, Wordpress Foundation have committed[24] to upholding the OSI's mission and Open Source Definition through the OSI Affiliate Agreement. Q Source: Frequently in news 13 In India, Rhododendrons are naturally found in A. B. C. D.

Uttarakhand, Sikkim and West Bengal West Bengal, Odisha, Madhya Pradesh and Chattisgarh Kerala, Karnataka and Tamil Nadu Gujarat and Mahrashtra

Correct Answer : A Answer Justification : Learning: It is found both in eastern and western Himalayas. Rhododendrons can be found in nearly all Himalayan states like NE India, Sikkim, WB, Uttarakhand, J&K, HP etc. It is a state symbol of Sikkim and Uttarakhand. The flowering plants with variegated flowers are fast disappearing from this region because of being ruthlessly felled down with dual purpose of obtaining fuel and construction of houses. Q Source: Improvisation on Qs asked on Eastern Himalayan vegetation in Past year UPSC papers

12

Mock Test 10 https://telegram.me/UPSCMaterials

https://telegram.me/FreeUPSCMaterials

https://telegram.me/MaterialforExam

14 IOWave18 is a mock exercise drill connected with tackling? A. B. C. D.

Piracy in Indian Ocean Tsunami rescue operations Traffic management in International Sea Lines of Communication None of the above

Correct Answer : B Answer Justification : Learning: India along with 23 other nations, would be participating in a major Indian ocean-wide tsunami mock exercise (drill) on September four and five, which would involve evacuation of thousands of people from coastal areas in over half a dozen states. The exercise, known as IOWave18, is being organised by the Intergovernmental Oceanographic Commission (IOC) of UNESCO, which coordinated the setting up of the Indian Ocean Tsunami Warning and Mitigation System (IOTWMS) in the aftermath of December 26, 2004 tsunami, an official release said here Monday. The Indian Tsunami Early Warning Centre (ITEWC), based out of the Indian National Centre for Ocean Information Services (INCOIS), Hyderabad, is an autonomous institution under the Union Ministry of Earth Sciences. "The IOWave18 exercise will simulate Indian Ocean countries being put in a tsunami warning situation and require the National Tsunami Warning Centre (NTWC), i.e., INCOIS in case of India, and the National and Local Disaster Management Offices (NDMO/LDMO) to implement their Standard Operating Procedures (SOPs)," it said. The purpose of exercise is to increase tsunami preparedness, evaluate response capabilities in each state and improve coordination throughout the region. The exercise would involve the evacuation of more than 1,25,000 people from the coastal communities of Odisha, Andaman and Nicobar Islands, Andhra Pradesh, Tamil Nadu, Puducherry, Maharashtra, West Bengal, Gujarat and Goa Q Source: https://www.business-standard.com/article/pti-stories/india-to-take-part-in-tsunami-mock-drill-on-se pt-4-5-118090301086_1.html 15 Consider the following capitals of North-eastern states of India. 1. Aizwal 2. Imphal 3. Kohima 4. Itanagar What is the correct order of these capitals from south to North? 13

Mock Test 10 https://telegram.me/UPSCMaterials

A. B. C. D.

https://telegram.me/FreeUPSCMaterials

https://telegram.me/MaterialforExam

1234 2314 3214 1342

Correct Answer : A Answer Justification : Justification:

Q Source: Map-based questions: India or South Asia 16 India is a member country of which of the following organizations? 1. East Asia Summit 2. ASEAN + 3 3. Asia-Pacific Economic Cooperation (APEC) Select the correct answer using the codes below. A. 1 only B. 1 and 3 only C. 2 and 3 only 14

Mock Test 10 https://telegram.me/UPSCMaterials

https://telegram.me/FreeUPSCMaterials

https://telegram.me/MaterialforExam

D. 1 and 2 only orrect Answer : A

Answer Justification : Justification: Statement 1: 16 countries in the East Asian, Southeast Asian, South Asian regions (including India), USA and RUSSIA are its members. Trade is an important focus for the summit. Statement 2: ASEAN+3 includes the 10 members of the Association of Southeast Asian Nations (Brunei Darussalam, Cambodia, Indonesia, Lao People's Democratic Republic, Malaysia, Myanmar, the Philippines, Singapore, Thailand, and Viet Nam) plus China, Japan, and Korea. It excludes India. So, 2 is incorrect. Statement 3: You can see APEC member states here. China has consistently blocked India’s bid to APEC membership. Details about APEC are covered in another question.

Q Source: http://www.insightsonindia.com/2018/09/03/insights-daily-current-affairs-03-september-2018/ 17 Gadgil and Kasturirangan committee setup some time ago and seen frequently in news are concerned with the conservation of which of these regions?

15

Mock Test 10 https://telegram.me/UPSCMaterials

A. B. C. D.

https://telegram.me/FreeUPSCMaterials

https://telegram.me/MaterialforExam

Western Ghats Eastern Ghats Northern Himalayas Eastern Himalayas

Correct Answer : A Answer Justification : Learning: The Gadgil committee defined the boundaries of the Western Ghats for the purposes of ecological management. It proposed that this entire area be designated as ecologically sensitive area (ESA). Within this area, smaller regions were to be identified as ecologically sensitive zones (ESZ) I, II or III based on their existing condition and nature of threat. It proposed to divide the area into about 2,200 grids, of which 75 per cent would fall under ESZ I or II or under already existing protected areas such as wildlife sanctuaries or natural parks. The committee proposed a Western Ghats Ecology Authority to regulate these activities in the area. Kasturirangan Committee: None of the six concerned states agreed with the recommendations of the Gadgil Committee, which submitted its report in August 2011. In August 2012, then Environment Minister constituted a High-Level Working Group on Western Ghats under Kasturirangan to “examine” the Gadgil Committee report in a “holistic and multidisciplinary fashion in the light of responses received” from states, central ministries and others. Its report revealed that of the nearly 1,750 responses it had examined, 81% were not in favour of the Gadgil recommendations. In particular, Kerala had objected to the proposed ban on sand mining and quarrying, restrictions on transport infrastructure and wind energy projects, embargos on hydroelectric projects, and inter-basin transfer of river waters, and also the complete ban on new polluting industries. Q Source: http://www.insightsonindia.com/2018/09/03/insights-daily-current-affairs-03-september-2018/ 18 The speed of tsunami waves depends on 1. Ocean depth 2. Distance from the source of the wave

16

Mock Test 10 https://telegram.me/UPSCMaterials

https://telegram.me/FreeUPSCMaterials

https://telegram.me/MaterialforExam

Which of the above is/are correct? A. 1 only B. 2 only C. Both 1 and 2 D. None

Correct Answer : A Answer Justification : Justification: Tsunamis are giant waves caused by earthquakes or volcanic eruptions under the sea. Out in the depths of the ocean, tsunami waves do not dramatically increase in height. But as the waves travel inland, they build up to higher and higher heights as the depth of the ocean decreases. The speed of tsunami waves depends on ocean depth rather than the distance from the source of the wave. Tsunami waves may travel as fast as jet planes over deep waters, only slowing down when reaching shallow waters. While tsunamis are often referred to as tidal waves, this name is discouraged by oceanographers because tides have little to do with these giant waves. Q Source: Additional Research on Chapter 5: 7th NCERT Geography 19 You will NOT find this salt or element dissolved in Ocean water. 1. Strontium 2. Borate 3. Vanadium Select the correct answer using the codes below. A. 1 only B. 2 and 3 only C. 3 only D. All of them can be found in Ocean water.

Correct Answer : D Answer Justification : Justification:

17

Mock Test 10 https://telegram.me/UPSCMaterials

https://telegram.me/FreeUPSCMaterials

https://telegram.me/MaterialforExam

Seawater contains more dissolved ions than all types of freshwater. However, the ratios of solutes differ dramatically. For instance, although seawater contains about 2.8 times more bicarbonate than river water, the percentage of bicarbonate in seawater as a ratio of all dissolved ions is far lower than in river water. Bicarbonate ions constitute 48% of river water solutes but only 0.14% for seawater. Differences like these are due to the varying residence times of seawater solutes; sodium and chloride have very long residence times, while calcium (vital for carbonate formation) tends to precipitate much more quickly. The most abundant dissolved ions in seawater are sodium, chloride, magnesium, sulfate and calcium. Q Source: Chapter 13: 11th NCERT Fundamentals of Physical Geography 18

Mock Test 10 https://telegram.me/UPSCMaterials

https://telegram.me/FreeUPSCMaterials

https://telegram.me/MaterialforExam

20 What is the Standard model of physics? 1. It describes how particles and forces interact. 2. It does not accept the fundamental forces of nature on a subatomic scale. Which of the above is/are correct? A. 1 only B. 2 only C. Both 1 and 2 D. None

Correct Answer : A Answer Justification : Background and Justification: In the world of particle physics, subatomic particles are difficult to observe because of their size. They are smaller than an atom and the wavelength of visible light, so the only way we can detect and observe their behavior is by smashing the atomic nucleus of particles together at intense speeds (close to the speed of light), which generates vast amounts of exotic particles that are only created at high energies. These collisions resemble the conditions physicists believe existed during the time of the big bang. Thanks to particle accelerators like the Large Hadron Collider, the Relativistic Heavy Ion Collider and the (now defunct) Tevatron circular particle accelerator, physicists have made a lot of progress in designing a “theory of everything.” This theory postulates how all the subatomic particles in the universe operates and how they interact to comprise the Universe as we know it. One of the most complete models that comes anywhere near producing a “theory of everything,” is the Standard Model of Fundamental Particles and Interactions, which describes how particles and forces interact. The standard model also includes an explanation for 3 of the 4 fundamental forces of nature on a subatomic scale. Q Source: http://www.insightsonindia.com/2018/08/29/insights-daily-current-affairs-29-august-2018/ 21 Which of these personalities were associated with Brahmo Samaj? 1. Debendranath Tagore 2. Keshub Chandra Sen 3. Pronob Bhattacharjee Select the correct answer using the codes below. A. 1 and 2 only B. 2 only 19

Mock Test 10 https://telegram.me/UPSCMaterials

https://telegram.me/FreeUPSCMaterials

https://telegram.me/MaterialforExam

C. 2 and 3 only D. 1 and 3 only

Correct Answer : A Answer Justification : Justification: Statement 1: In 1839, he along with his friends started the Tattwaranjini Sabha which was later renamed to Tattwabodhini Sabha. At this time the affairs of the Samaj were being managed by the Tattwabodhini Sabha and Pt. Iswar Chandra Vidyasagar was its secretary. In 1859 the Tattwabodhini Sabha was abolished and the propagatroy work was taken up by the Samaj - the printing press, library and other properties were made over to the Trustees of the same. Vidyasagar could not agree with the changes introduced and resigned from his post. Statement 2: He joined the Brahmo Samaj in 1857 by privately signing the Brahmo covenant and took to studying mental and moral philosophy. During this time in 1860, the Sangat Sabha was established - which was a society of fellow believers to promote mutual spiritual intercourse amongst its members. T Q Source: Improvisation on Brahmo Samaj Qs asked in Past year UPSC papers 22 Consider the following statements. Assertion (A): Steep waves in oceans originate from faraway places, possibly from another hemisphere, and travel long distances in the ocean. Reason (R): The maximum wave height is determined by the strength of the wind. In the context of the above, which of these is correct? A. A is correct, and R is an appropriate explanation of A. B. A is correct, but R is not an appropriate explanation of A. C. A is incorrect, but R is correct. D. Both A and R are incorrect. orrect Answer : C

Answer Justification : Justification: A wave’s size and shape reveal its origin. Steep waves are fairly young ones and are probably formed by local wind. Slow and steady waves originate from far away places, possibly from another hemisphere. The maximum wave height is determined by the strength of the wind, i.e. how long it blows and the area over which it blows in a single direction. 20

Mock Test 10 https://telegram.me/UPSCMaterials

https://telegram.me/FreeUPSCMaterials

https://telegram.me/MaterialforExam

Waves travel because wind pushes the water body in its course while gravity pulls the crests of the waves downward. th

Q Source: Chapter 14: 11 NCERT Fundamentals of Physical Geography 23 The Pradhan Mantri Awas Yojana (Urban) Programme seeks to address the housing requirement of urban poor including slum dwellers through 1. Slum rehabilitation of Slum Dwellers with participation of private developers using land as a resource. 2. Promotion of Affordable Housing for weaker section through credit linked subsidy. 3. Affordable Housing in Partnership with Public & Private sectors. 4. Subsidy for beneficiary-led individual house construction or enhancement. Select the correct answer using the codes below. A. 1, 2 and 3 only B. 2, 3 and 4 only C. 1 and 4 only D. 1, 2, 3 and 4 only orrect Answer : D

Answer Justification : Justification: The Pradhan Mantri Awas Yojana (Urban) Programme launched by the Ministry of Housing and Urban Poverty Alleviation (MoHUPA), in Mission mode envisions provision of Housing for All by 2022, when the Nation completes 75 years of its Independence. The beneficiaries are poor and people living under EWS and LIG categories in the country. The scheme is divided into three phases. In the first phase, a total of 100 cities will be covered from April 2015 to March 2017. In phase two, 200 cities will be covered from April 2017 to March 2019. In the third phase, the leftover cities will be covered from April 2019 to March 2022. The government is providing an interest subsidy of 6.5% on housing loans which can be availed by beneficiaries for 15 years from start of loan date. The government will grant Rs 1 lakh to all the beneficiaries of the scheme. In addition, Rs 1.5 lakh will be given to all eligible urban poor who want to construct their houses in urban areas or plan to go for renovation in their existing houses. One can also avail loans under this scheme to build toilets in existing houses. Q Source: http://www.insightsonindia.com/2018/08/25/insights-daily-current-affairs-25-august-2018/ 24 “The Slave Route project” is an initiative of

21

Mock Test 10 https://telegram.me/UPSCMaterials

A. B. C. D.

https://telegram.me/FreeUPSCMaterials

https://telegram.me/MaterialforExam

UNESCO United Nations Development Programme World Economic Forum United Nations Commissioner for Refugees

Correct Answer : A Answer Justification : Learning: The Slave Route Project is a UNESCO initiative that was officially launched in 1994 in Ouidah, Benin. It is rooted in the mandate of the Organization, which believes that ignorance or concealment of major historical events constitutes an obstacle to mutual understanding, reconciliation and cooperation among peoples. The project breaks the silence surrounding the slave trade and slavery that has affected all continents and caused great upheavals that have shaped our modern societies. In studying the causes, the modalities and the consequences of slavery and the slave trade, the project seeks to enhance the understanding of diverse histories and heritages stemming from this global tragedy. It wishes to highlight the global transformations and cultural interactions that have resulted from this history; Q Source: http://www.unesco.org/new/en/social-and-human-sciences/themes/slave-route/ 25 Consider the following statements. 1. The shape of bays and estuaries along a coastline can also magnify the intensity of tides. 2. When tide is channelled between islands or into bays and estuaries it becomes tidal current. Which of the above is/are correct? A. 1 only B. 2 only C. Both 1 and 2 D. None

Correct Answer : C Answer Justification : Justification: Statement 1: On the surface of the earth, the horizontal tide generating forces are more important than the vertical forces in generating the tidal bulges. The tidal bulges on wide continental shelves have greater height. In the open ocean tidal currents are relatively weak. When tidal bulges hit the mid-oceanic islands they become low.

22

Mock Test 10 https://telegram.me/UPSCMaterials

https://telegram.me/FreeUPSCMaterials

https://telegram.me/MaterialforExam

Funnel-shaped bays greatly change tidal magnitudes. Statement 2: Tidal currents occur in conjunction with the rise and fall of the tide. The vertical motion of the tides near the shore causes the water to move horizontally, creating currents. When a tidal current moves toward the land and away from the sea, it “floods.” When it moves toward the sea away from the land, it “ebbs.” Tidal currents are the only type of current affected by the interactions of the Earth, sun, and moon. Q Source: Chapter 14: 11th NCERT Fundamentals of Physical Geography 26 With reference to the Inter State Transmission System, consider the following statements. 1. It is a system for the conveyance of electricity by means of a main transmission line from the territory of one State to another State. 2. ISTS may be owned by Central Transmission Utility, State Utility or even Independent Power Transmission Companies. 3. ISTS tariff is determined as per the CERC (Terms and Conditions of Tariff) regulations. Select the correct answer using the codes below. A. 2 only B. 1, 2 and 3 only C. 1 only D. 1 and 3 only orrect Answer : B

Answer Justification : Justification: ISTS helps in: The conveyance of electricity across the territory of an intervening State as well as conveyance within the State which is incidental to such inter-state transmission of energy The transmission of electricity within the territory of State on a system built, owned, operated, maintained or controlled by CTU; ISTS may be owned by Central Transmission Utility, State Utility, Independent Power Transmission Companies ISTS tariff is determined as per the CERC (Terms and Conditions of Tariff), Regulations, 2009. ISTS licensee submits tariff petition for approval by CERC. Government has nominated state-run Power Grid Corp for developing power transmission lines connecting solar parks and Inter-State Transmission System (ISTS), which has led to some private companies crying foul that competitive bidding route should have been followed. 23

Mock Test 10 https://telegram.me/UPSCMaterials

https://telegram.me/FreeUPSCMaterials

https://telegram.me/MaterialforExam

The power transmission lines for connecting solar parks and ISTS in the country will be set up by the Power Grid Corporation. Q Source: https://www.business-standard.com/article/companies/powergrid-to-set-up-lines-to-link-solar-parkswith-ists-115092000127_1.html https://www.business-standard.com/article/economy-policy/rs-30-000-crore-bond-opportunity-in-pow er-transmission-crisil-117122001030_1.html 27 The petromoneda, is a cryptocurrency developed by the government of A. B. C. D.

Peru Venezuela Brazil Argentina

Correct Answer : B Answer Justification : Learning: Announced in December 2017, it is backed by the country's oil and mineral reserves, and it is intended to supplement Venezuela's plummeting bolívar fuerte currency, as a means of circumventing U.S. sanctions and accessing international financing. On August 20th 2018 the bolívar soberano (Sovereign Bolivar) was introduced, with the government stating it would be linked to the petro coin value The petro is a token based on the NEM blockchain. The design of the petro by the Venezuelan government has been controversial, with white paper changing by the day even after the petro's pre-sale. Petros are "pre-mined" by the Venezuelan government, meaning that new tokens cannot be created after the issuance. Members of the Venezuelan Ministry of University Education, Science and Technology allegedly under advisory of the Russian government designed the petro to circumvent United States sanctions, with Russian president Vladimir Putin devising official involvement. Q Source: The currency was launched in February, 2018 28 Consider the following statements. 1. The Nag missile is the most affordable Indian third generation "fire-and-forget" anti-tank guided missile costing under $5,000 USD per unit. 2. NAMICA (Nag Missile Carrier) is a tank destroyer built for the army. 3. HeliNa (Helicopter-launched Nag) has an intercontinental ballistic missile range. 24

Mock Test 10 https://telegram.me/UPSCMaterials

https://telegram.me/FreeUPSCMaterials

https://telegram.me/MaterialforExam

Select the correct answer using the codes below. A. 1 and 2 only B. 2 only C. 2 and 3 only D. 1 and 3 only

Correct Answer : B Answer Justification : Justification: Statement 1: The Nag missile is an Indian third generation "fire-and-forget" anti-tank guided missile. It is an all weather, top attack missile with a range of 3 to 7 km. It is being developed by India's Defence Research and Development Organisation (DRDO) under the Integrated Guided Missile Development Program and is manufactured by Bharat Dynamics Limited. The Nag missile costs about $500,000 USD per unit and has a single-shot hit probability of 0.77. As of 2017, the Nag missile is solely operated from the NAMICA ATGM carrier, a tank destroyer based on the BMP-2 platform. The Nag missile has been described as "long-delayed" and as of 2018 it has not yet entered service. Statement 2: NAMICA (Nag Missile Carrier) is a tank destroyer built for the army. It is equipped with a thermal imager for target acquisition. NAMICA is a modified BMP-2 IFV produced as "Sarath" in India. The carrier weights 14.5 tonnes in full combat load and is capable of moving 7 km/h in water. The carriers are capable of carrying 12 missiles with 8 in ready-to-fire mode. The NAMICA carrier was put through transportation trials covering 155 km during 2008 summer trials. Statement 3: HeliNa, (Helicopter-launched Nag) with a range of 7–8 km, launched from twin-tube stub wing-mounted launchers on board the armed HAL Dhruv and HAL Light Combat Helicopter produced by state-owned Hindustan Aeronautics Ltd (HAL). It will be structurally different from the Nag. Q Source: https://timesofindia.indiatimes.com/topic/Nag-(HeliNa)-missile/news 29 Chang'e, recently in news is a A. Interstellar mission of NASA B. Ongoing series of robotic Moon missions by the China National Space Administration (CNSA) C. A collaborative deep-sea mining endeavour of the People’s Republic of China, Russia and Pakistan D. A quantum satellite

Correct Answer : B 25

Mock Test 10 https://telegram.me/UPSCMaterials

https://telegram.me/FreeUPSCMaterials

https://telegram.me/MaterialforExam

Answer Justification : Learning: Chang-o, originally known as Heng'e, is the Chinese goddess of the Moon. She is the subject of several legends in Chinese mythology. Chang'e has been the namesake of the Chinese Lunar Exploration Program. Chang'e, is an ongoing series of robotic Moon missions by the China National Space Administration (CNSA). The program incorporates lunar orbiters, landers, rovers and sample return spacecraft, launched using Long March rockets. Launches and flights are monitored by a Telemetry, Tracking, and Command (TT&C) system, which uses 50-metre (160-foot) radio antennas in Beijing and 40-metre (130-foot) antennas in Kunming, Shanghai, and Ürümqi to form a 3,000-kilometre (1,900-mile) VLBI antenna. A proprietary ground application system is responsible for downlink data reception. Q Source: https://www.space.com/41613-china-moon-far-side-mission-change-4-details.html 30 The currents that mainly flow in the Northern Hemisphere are 1. West Wind drift 2. Gulf Stream 3. Canaries current Select the correct answer using the codes below. A. 1 only B. 2 and 3 only C. 1 and 3 only D. 1, 2 and 3

Correct Answer : B Answer Justification : Justification: This figure from NCERT clearly shows all such currents.

26

Mock Test 10 https://telegram.me/UPSCMaterials

https://telegram.me/FreeUPSCMaterials

https://telegram.me/MaterialforExam

Q Source: Figure 14.3 from Chapter 14: 11th NCERT Fundamentals of Physical Geography 31 Consider the following statements. 1. TRIFED has developed a system of sourcing products of art and craft made by empanelled tribal suppliers. 2. TRIFED has been marketing tribal products through its own shops called "TRIBES India”. Which of the above is/are correct? A. 1 only B. 2 only C. Both 1 and 2 D. None

Correct Answer : C Answer Justification : Justification: The Tribal Cooperative Marketing Development Federation of India, (TRIFED), under the Ministry of Tribal Affairs, is engaged in marketing development of tribal products including art and craft items. TRIFED has developed a system of sourcing products of art and craft made by empanelled tribal suppliers. The suppliers comprise of individual tribal artisans, tribal SHGs, Organisations/ Agencies/NGOs working with tribals, who are empanelled as suppliers of TRIFED as per the guidelines for empanelment of suppliers. The products procured by TRIFED are then sold through 27

Mock Test 10 https://telegram.me/UPSCMaterials

https://telegram.me/FreeUPSCMaterials

https://telegram.me/MaterialforExam

its marketing platform of "Tribes India". A e-commerce (electronic commerce) website - www.tribesindia.com has hence been made available for sale of all its products. Q Source: https://timesofindia.indiatimes.com/business/india-business/in-sal-leaves-auto-companies-may-find-acleanser/articleshow/65663620.cms 32 Consider the following statements about Bombay Natural History Society (BNHS). 1. It is an autonomous governmental agency under the Wildlife Trust of India. 2. BNHS is the partner of BirdLife International in India. 3. It has been designated as a ‘Scientific and Industrial Research Organisation’ by the Department of Science and Technology. Select the correct answer using the codes below. A. 1 and 2 only B. 2 and 3 only C. 1 and 3 only D. 1, 2 and 3

Correct Answer : B Answer Justification : Justification: Bombay Natural History Society, founded in 1883, is one of the largest nongovernmental organisations in India engaged in conservation and biodiversity research. BNHS is the partner of BirdLife International in India. It has been designated as a ‘Scientific and Industrial Research Organisation’ by the Department of Science and Technology. The BNHS logo is the great hornbill. Internet of Birds: IT consultancy firm Accenture and the Bombay Natural History Society have developed Internet of Birds platform that identifies bird species found in India using Artificial Intelligence technology, including machine learning and computer vision, from digital photos that are uploaded by the public. Q Source: http://www.insightsonindia.com/2018/08/27/insights-daily-current-affairs-27-august-2018/ 33 Catalonia, was recently in news due to which of the following reasons? A. Intended separation from Spain B. Recording the highest GDP growth among all major cities of the World C. Running completely on Solar power 28

Mock Test 10 https://telegram.me/UPSCMaterials

https://telegram.me/FreeUPSCMaterials

https://telegram.me/MaterialforExam

D. Declared a tax haven

Correct Answer : A Answer Justification : Learning: Catalonia has its own distinct language, was granted autonomy under Spain’s 1978 Constitution adopted three years after the death of longtime dictator Francisco Franco. In 2006, a statute granting even greater powers to the northwestern region, boosting its financial clout, was approved by the Spanish and Catalan parliaments. And in a referendum at the time, over 73% of voters in Catalonia approved it. But in 2010 Spain’s Constitutional Court struck down several articles of the charter, among them attempts to place the distinctive Catalan language above Spanish in the region and a clause describing the region as a “nation”. The ruling sparked a rise in support for independence in Catalonia, which is home to some 7.5 million people and accounts for about one-fifth of the Spanish economy. How would a secession affect the Spanish economy? The Catalan region has long been the industrial heartland of Spain, with textile and shipbuilding, and more recently, finance, services, and technology. Barcelona has a thriving start-up culture, and plays host to the annual Mobile World Congress, where the bleeding edge of technology is on display. Catalonia is one of the wealthiest regions of Spain. It accounts for 20.07% of the Spanish GDP. Secession would therefore cost Spain almost a fifth of its economic output, and trigger a row on how to carve up the €836 billion of national debt. Q Source: http://www.insightsonindia.com/2018/09/04/insights-daily-current-affairs-04-september-2018/ 34 Consider the following statements. 1. Sherpas are career diplomats or senior government officials appointed by the leaders of their countries. 2. Sherpas participate in a series of pre-Summit consultations to help negotiate their leaders’ positions. Which of the above is/are correct? A. 1 only B. 2 only C. Both 1 and 2 D. None

Correct Answer : C 29

Mock Test 10 https://telegram.me/UPSCMaterials

https://telegram.me/FreeUPSCMaterials

https://telegram.me/MaterialforExam

Answer Justification : Justification: A Sherpa is a personal representative of the leader of a member country at an international Summit meeting such as the G8, G20 or the Nuclear Security Summit. The term is derived from the Nepalese Sherpa people, who serve as guides for mountaineers in the Himalayas. The Sherpa engages in planning, negotiation and implementation tasks through the Summit. They coordinate the agenda, seek consensus at the highest political levels, and participate in a series of pre-Summit consultations to help negotiate their leaders’ positions. There is only one Sherpa per Summit for each member country; he/she is assisted by several sous Sherpas. Sherpas meet much before the start of the Summit to iron out differences on various issues. For the 2016 G20 Summit, the first meeting of Sherpas was held in January 2016, followed by a meeting each in April and June. At the G20 Summit, work progresses through broadly two channels: the Finance Track and Sherpas’ Track. Towards the end of the process, the Sherpas, along with the Finance Track representatives, prepare the Leaders’ “Declaration” or “Communique”, which is the final outcome of the G20 Summit. The Sherpas’ Track involves technical and policy analyses by working groups comprising officials from each member country and international organisations. It focuses on development-oriented issues such as agriculture, fighting corruption, employment, etc. https://indianexpress.com/article/explained/g20-meet-what-role-does-the-sherpa-play-in-the-negotiat ions-3015461/ 35 Arrange the following Gulfs of South Asia from North to South. 1. Gulf of Martaban 2. Gulf of Khambat 3. Gulf of Mannar 4. Gulf of Kutch Select the correct answer using the codes below. A. 1243 B. 4213 C. 2143 D. 1324

Correct Answer : B

30

Mock Test 10 https://telegram.me/UPSCMaterials

https://telegram.me/FreeUPSCMaterials

https://telegram.me/MaterialforExam

Answer Justification : Justification: Gulf of Kutch lies northwards to Gulf of Khambat in Gujarat. So, 2 must come after 4. Gulf of Martaban is near Myanmar. Gulf of Mannar is near Tamilnadu. So, 3 must come in the end and correct answer must be B.

Q Source: Map based question: Region: South Asia 36 This bacterium is associated with food stored for long periods of time in the refrigerator as it is ubiquitous and has the ability to grow slowly even at low temperatures. It is an important cause of meningitis. The bacterium is? A. B. C. D.

Listeria monocytogenes Bacillus cereus S. pyogenes Mycoplasma genitalium

Correct Answer : A Answer Justification : Learning: It can grow and reproduce inside the host's cells and is one of the most virulent foodborne pathogens. It is capable of surviving in the presence or absence of oxygen.

31

Mock Test 10 https://telegram.me/UPSCMaterials

https://telegram.me/FreeUPSCMaterials

https://telegram.me/MaterialforExam

A person with listeriosis usually has fever and muscle aches, sometimes preceded by diarrhea or other gastrointestinal symptoms. Almost everyone who is diagnosed with listeriosis has "invasive" infection, in which the bacteria spread beyond the gastrointestinal tract. Pregnant women and older adults are at higher risks. Q Source: Additional Research: UPSC Prelims questions on Alfatoxins 37 Istanbul is located at the junction of A. B. C. D.

Black Sea and Mediterranean Sea Red Sea and Caspian Sea Caspian Sea and Black Sea Mediterranean Sea and Caspian Sea

Correct Answer : A Answer Justification : Learning: Istanbul is a transcontinental city in Eurasia, straddling the Bosphorus strait (which separates Europe and Asia) between the Sea of Marmara and the Black Sea. Istanbul's strategic position on the historic Silk Road, rail networks to Europe and the Middle East, and the only sea route between the Black Sea and the Mediterranean have produced a cosmopolitan populace.

32

Mock Test 10 https://telegram.me/UPSCMaterials

https://telegram.me/FreeUPSCMaterials

https://telegram.me/MaterialforExam

Q Source: http://www.bbc.co.uk/news/world-europe-38276794 + Heart of Asia Conference traces its origin to Istanbul 38 The education conference held at Wardha in October 1937 is notable for 1. Promoting free and compulsory education for an individual throughout his life 2. Endorsing Gandhiji's proposals for 'basic education' through the vernacular medium and manual productive work 3. Motivating teachers to work without any payment for the welfare of nation at large. Select the correct answer using the codes below. A. 1 and 2 only B. 2 only C. 2 and 3 only D. 1 and 3 only

Correct Answer : B Answer Justification : Justification: In 1937, Gandhiji had published an article in the Harijan, based upon which, an all India National Education Conference was held in 1937 - Wardha Educational Conference. The 33

Mock Test 10 https://telegram.me/UPSCMaterials

https://telegram.me/FreeUPSCMaterials

https://telegram.me/MaterialforExam

president of this conference was Gandhi. The resolutions passed were as follows: Free and compulsory education to be provided for 7 years at a nationwide scale. Mother tongue should be the medium of instruction. Throughout this period of 7 years, the education should be around some forms of manual and productive work and for this purpose a handicraft must be chosen, based upon the environment of the child. This system would generate the remuneration of the teachers. So, 3 is wrong. Also, a Basic Education Committee was setup under Dr. Zakir Hussain following Wardha conference to formulate the scheme of the basic education. The aim of the basic education was to develop the qualities of the ideal citizenship and more aspect should be give to the Indian culture than the literacy. There should be NO PLACE for English in the curriculum. There was no place for religious education in this scheme. Q Source: Improvisation on Woods Despatch asked in Past year UPSC papers 39 As per the D.K. Basu Guidelines laid down by the Supreme Court 1. The arrest memo should be counter-signed by the person arrested. 2. The person arrested, detained or being interrogated has a right to inform a relative, friend or wellwisher. 3. The Police Officer cannot arrest a person who is diseased or suffering from a body ailment. 4. No agency other than the police will have the rights of interrogating the accused. Select the correct answer using the codes below. A. 1 and 2 only B. 2 and 4 only C. 1, 3 and 4 only D. 1, 2 and 3 only orrect Answer : A

Answer Justification : Justification: Given the present trend of UPSC, you should be prepared to face such slightly detailed questions on SC guidelines and judgments, both of the past and in the recent times.

34

Mock Test 10 https://telegram.me/UPSCMaterials

https://telegram.me/FreeUPSCMaterials

https://telegram.me/MaterialforExam

The Supreme Court of India has laid down specific requirements and procedures that the police and other agencies have to follow for the arrest, detention and interrogation of any person. These are known as the D.K. Basu Guidelines and some of these include: The police officials who carry out the arrest or interrogation should wear clear, accurate and visible identification and name tags with their designations; A memo of arrest should be prepared at the time of arrest and should include the time and date of arrest. It should also be attested by at least one witness who could include a family member of the person arrested. The arrest memo should be counter-signed by the person arrested. When a friend or relative lives outside the district, the time, place of arrest and venue of custody must be notified by police within 8 to 12 hours after arrest. The person arrested must be made aware of his right to have someone informed of his arrest or detention as soon as he is put under arrest or is detained. Q Source: Revision questions: Previous Test Syllabus 40 In the practice of Organic Farming 1. No genetic modification is done to increase the yield of the crop. 2. No manures are used to modify the natural properties of the soil. 3. No pesticides or weedicides are used allowing the food chain to work naturally at the farm level. Select the correct answer using the codes below. A. 1 and 3 only B. 2 and 3 only C. 1 only D. 1, 2 and 3

Correct Answer : C Answer Justification : Justification: Organic farming is a system which avoids or largely excludes the use of synthetic inputs (such as fertilizers, pesticides, hormones, feed additives etc) and to the maximum extent feasible rely upon crop rotations, crop residues, animal manures, off-farm organic waste, mineral grade rock additives and biological system of nutrient mobilization and plant protection.

35

Mock Test 10 https://telegram.me/UPSCMaterials

https://telegram.me/FreeUPSCMaterials

https://telegram.me/MaterialforExam

Statement 1: It avoids the use of GM seeds to reduce complications on the field and uses naturally obtained seeds. Statement 2: It does not use chemical fertilizers, but uses organic manures to boost the productivity and nutrient base of the soil. So, 2 is incorrect. You should read such statements carefully. Statement 3: Similarly, it uses natural methods of pest control, and natural weedicides and herbicides, and not chemical ones. So, statement 3 is incorrect. Q Source: Revision questions: Previous Test Syllabus 41 Consider the following statements. 1. Scheme of Ashram Schools in Tribal Areas intends to provide residential schools for STs to increase the literacy rate among the tribal students. 2. Scheme for Strengthening Education among ST Girls in Low Literacy Districts aims to bridge the gap in literacy levels between the general female population and tribal women, through facilitating 100% enrolment particularly in naxal affected areas. Which of the above is/are correct? A. 1 only B. 2 only C. Both 1 and 2 D. None

Correct Answer : C Answer Justification : Justification: Statement 1: Scheme of Ashram Schools in Tribal Areas: The objective of the scheme is to provide residential schools for STs to increase the literacy rate among the tribal students and to bring them at par with other population of the country. Under the scheme, State Governments are eligible for 100% central share for construction of all Girls’ Ashram Schools and also for construction of Boys’ Ashram Schools in naxal affected areas. The funding pattern for the other Boys’ Ashram Schools is on 50:50 basis. Statement 2: Scheme for Strengthening Education among ST Girls in Low Literacy Districts:The scheme aims to bridge the gap in literacy levels between the general female population and tribal women, through facilitating 100% enrolment of tribal girls in the identified Districts or Blocks, more particularly in naxal affected areas and in areas inhabited by Primitive Tribal Groups (PTGs), and reducing drop-outs at the elementary level by creating the required ambience for education. Improvement of the literacy rate of tribal girls is essential to enable them to participate effectively in and benefit from, socio-economic development. Q Source: Additional Research in tribal educational covered in Chapter 5: NCERT 7th: Social and Political Life 42 Consider the following about the role of Dr. B.R. Ambedkar. 36

Mock Test 10 https://telegram.me/UPSCMaterials

https://telegram.me/FreeUPSCMaterials

https://telegram.me/MaterialforExam

1. He was the founder of Republican Party of India and Swatantra Party. 2. He was the law minister in the first cabinet of post-independent India. 3. He once acted as the President of the Bharatiya Jansangh. Select the correct answer using the codes below. A. 1 and 3 only B. 2 only C. 1 and 2 only D. 1, 2 and 3

Correct Answer : B Answer Justification : Justification: Statement 1: Swantantra party was founded by Kanhaiyalal Maniklal Munshi. Ambedkar founded the Republican Party of India. Statement 2: He also served as the Chairman of the Drafting Committee of the Constitution. Statement 3: It was Shyama Prasad Mukherjee who was Active in Hindu Mahasabha and later became the founder President of Bharatiya Jansangh (now BJP). Q Source: Revision: Page 48: Democratic Politics - I 43 The term “Socialist” in the Preamble of the Constitution implies 1. Wealth should be generated socially and should be shared equitably by the society. 2. Government should regulate and facilitate the equitable ownership of factors of production to achieve desired socio-economic objectives. Which of the above is/are correct? A. 1 only B. 2 only C. Both 1 and 2 D. None

Correct Answer : C Answer Justification : Justification & Learning: The Indian brand of socialism is a ‘democratic socialism’ and not a ‘communistic socialism’ (also known as ‘state socialism’) which involves the nationalisation of all means of production and distribution and the abolition of private property. Democratic socialism, on the other hand, holds faith in a ‘mixed economy’ where both public and private sectors co-exist side by side.

37

Mock Test 10 https://telegram.me/UPSCMaterials

https://telegram.me/FreeUPSCMaterials

https://telegram.me/MaterialforExam

As the Supreme Court says, ‘Democratic socialism aims to end poverty, ignorance, disease and inequality of opportunity. Indian socialism is a blend of Marxism and Gandhism, leaning heavily towards Gandhian socialism’. The new economic policy (1991) of liberalisation, privatisation and globalisation has, however, diluted the socialist credentials of the Indian State. Q Source: Chapter 3: Democratic Politics - I 44 With reference to a Banking Ombudsman, consider the following statements. 1. Banking ombudsman is a quasi judicial authority, created to resolve customer complaints against banks relating to certain services provided by them. 2. He is appointed by the Reserve Bank of India. 3. Private banks do not have the mechanism of an ombudsman presently. Select the correct answer using the codes below. A. 1 only B. 1 and 2 only C. 2 and 3 only D. 1 and 3 only

Correct Answer : B Answer Justification : Justification: The Ombudsman is a senior official, who has been appointed by the Reserve Bank of India to address grievances and complaints from customers, pertaining deficiencies in banking services. It covers all kinds of banks including public sector banks, Private banks, Rural banks as well as cooperative banks. The Internal Ombudsman Scheme of 2018 mandates banks to grant a fixed term of three to five years, which cannot be renewed, to the IO. The IO can be removed only with prior approval from RBI. The remuneration would have to be decided by the customer sub-committee of the board and not by any individual. The Reserve Bank of India has tightened the banking ombudsman scheme with the objective to strengthen the grievance redressal mechanism for customers. New guidelines: The banking regulator has asked all commercial banks having 10 or more banking outlets to have an independent internal ombudsman (IO) to review customer complaints that are either partly or fully rejected by the banks. The IO shall, inter alia, examine customer complaints which are in the nature of deficiency in 38

Mock Test 10 https://telegram.me/UPSCMaterials

https://telegram.me/FreeUPSCMaterials

https://telegram.me/MaterialforExam

service on the part of the bank, that are partly or wholly rejected by the bank. As banks should internally escalate complaints that are not fully redressed to their respective IOs before conveying the final decision to the complainant, customers need not approach the IO directly. Internal Ombudsman Scheme: The Internal Ombudsman Scheme of 2018 mandates banks to grant a fixed term of three to five years, which cannot be renewed, to the IO. The IO can be removed only with prior approval from RBI. The remuneration would have to be decided by the customer sub-committee of the board and not by any individual. The Ombudsman Scheme of 2018 covers appointment/tenure, roles and responsibilities, procedural guidelines and oversight mechanism for the IO, among others. The implementation of IO Scheme 2018 will be monitored by the bank’s internal audit mechanism apart from regulatory oversight by RBI. Q Source: http://www.insightsonindia.com/2018/09/04/insights-daily-current-affairs-04-september-2018/ 45 The shoreline of the this Sea is the lowest dry land on Earth A. B. C. D.

Red Sea Dead Sea Caspian Sea North Sea

Correct Answer : B Answer Justification : Justification: The lowest land area is the shoreline of the Dead Sea Depression in Israel, Jordan and Syria. It is approximately 413 meters or 1355 feet below sea level. The largest below-sea-level depression by surface area is the Caspian Depression of Kazakhstan and Russia. It has an area of approximately 200,000 square kilometers or 77,000 square miles of land below sea level. Most major depressions are associated with tectonic plate boundaries. They form when converging plates deform or when spreading centers open. A few are volcanic in origin. Most of these depressions are in the northern hemisphere for a good reason: this is where most of the land is and where most of the plate boundaries occur. Many are found in the vicinity of where the African, Arabian and Eurasian plates meet. Most are found in hot desert regions of the planet where high evaporation rates prevent them from filling with water. A few are found in temperature climes. 39

Mock Test 10 https://telegram.me/UPSCMaterials

https://telegram.me/FreeUPSCMaterials

https://telegram.me/MaterialforExam

th

Q Source: Additional Research on Chapter 5: 7 NCERT Geography 46 Consider the following statements. 1. Mauritius is the top source of FDI in India. 2. FDI constitutes the majority of investment in infrastructure in India. 3. Infrastructure sector has received the least amount of FDI investment in the last one decade. Select the correct answer using the codes below. A. 1 only B. 1 and 2 only C. 1, 2 and 3 D. 1 and 3 only

Correct Answer : A Answer Justification : Justification: S1: As per the latest data by RBI, Mauritius was the top source of foreign direct investment (FDI) into India in 2017-18 followed by Singapore. The total FDI in FY 18 stood at $37.36 billion in financial year which was marginal rise over $36.31 billion recorded in the previous fiscal 2016-17. S2: Foreign direct investment (FDI) in India is a major monetary source for economic development in India, but constitute less than 2% of Gross Capital Formation (GCF). Foreign companies invest directly in fast growing private Indian businesses to take benefits of cheaper wages and changing business environment of India. Infrastructure 10% of India's GDP is based on construction activity. Indian government has invested $1 trillion on infrastructure from 2012–2017. 40% of this $1 trillion had to be funded by private sector. 100% FDI under automatic route is permitted in construction sector for cities and townships Q Source: https://timesofindia.indiatimes.com/business/india-business/mauritius-largest-source-of-fdi-in-india-s ays-rbi/articleshow/62571261.cms 47 With reference to India Post Payments Bank (IPPB), consider the following statements. 1. India Post Payments Bank has been allowed to link several of its postal savings bank (PSB) accounts with its accounts. 2. India Post Payments Bank cannot offer any interest rate on savings accounts. 3. It can accept demand deposits. 4. It can issue both ATM/debit cards and credit cards. Select the correct answer using the codes below. 40

Mock Test 10 https://telegram.me/UPSCMaterials

A. B. C. D.

https://telegram.me/FreeUPSCMaterials

https://telegram.me/MaterialforExam

1 and 3 only 2 and 4 only 1, 3 and 4 only 1, 2 and 4 only

Correct Answer : A Answer Justification : Justification: The India Post Payments Bank (IPPB) is a public sector company under the department of posts and ministry of communication with a 100 per cent equity of the government of India, and governed by the Reserve Bank of India (RBI). IPPB will focus on providing banking and financial services to people in rural areas, by leveraging the reach of 1.55 lakh post office branches. The government aims to link all the 1.55 lakh post offices to the India Post Payments Bank system by 31 December, 2018. It started operations on 30 January, 2017, by opening two pilot branches, one at Raipur and the other at Ranchi. S2 India Post Payments Bank will offer 4 per cent interest rate on savings accounts. India Post Payments Bank will offer a range of products such as savings and current accounts, money transfer, direct benefit transfers, bill and utility payments, and enterprise and merchant payments. India Post Payments Bank has been allowed to link around 17 crore postal savings bank (PSB) accounts with its accounts. SLR: Apart from amounts maintained as Cash Reserve Ratio (CRR) with the Reserve Bank on its outside demand and time liabilities, it will be required to invest minimum 75% of its “demand deposit balances” in Statutory Liquidity Ratio(SLR) eligible Government securities/treasury bills with maturity up to one year and hold maximum 25% in current and time/fixed deposits with other scheduled commercial banks for operational purposes and liquidity management. S3 and S4: They can accept demand deposits. They can issue ATM/debit cards but not credit cards because payment banks cannot offer credit cards. http://www.insightsonindia.com/2018/09/01/insights-daily-current-affairs-01-september-2018/ 48 High tides can be helpful in 1. Sea navigation 2. Fishing 3. Energy generation Select the correct answer using the codes below. A. 1 only 41

Mock Test 10 https://telegram.me/UPSCMaterials

https://telegram.me/FreeUPSCMaterials

https://telegram.me/MaterialforExam

B. 2 and 3 only C. 1, 2 and 3 D. 1 and 3 only

Correct Answer : C Answer Justification : Justification: S1 Tides affect the depth and currents in and around coastal areas. Ships may need to navigate the waters during high tide in some areas or risk running aground. Pilots take into consideration the water level, width of channels and direction of the water flow to determine the best time to travel. Pilots may choose to travel when tides are at ebb in order to get tall loads under bridges. S2 Fish may concentrate during ebb tides. Commercial fishermen follow the tides and learn to fish during levels of highest concentration to improve their economic investment and to make more efficient use of their time. Recreational fishermen may also fish during ebb tides because the concentrations of smaller fish attract the larger trophy fish. The rise and fall of water due to tides is being used to generate electricity in some places. Q Source: Chapter 5: 7th NCERT Geography 49 Consider the following statements. 1. Generally, the warm ocean currents originate near the equator and move towards the poles. 2. The Labrador Ocean current is cold current while the Gulf Stream is a warm current. 3. Warm currents bring about warm temperature over land surface. 4. The areas where the warm and cold currents meet provide some of the best fishing grounds of the world. Select the correct answer using the codes below. A. 1, 3 and 4 only B. 2, 3 and 4 only C. 1 and 2 only D. 1, 2, 3 and 4

Correct Answer : D Answer Justification : Justification: Ocean currents are streams of water flowing constantly on the ocean surface in definite directions. The ocean currents may be warm or cold. Generally, the warm ocean currents originate near the equator and move towards the poles due to the temperature differential between these two regions. S2 and S3 The cold currents carry water from polar or higher latitudes to tropical or lower 42

Mock Test 10 https://telegram.me/UPSCMaterials

https://telegram.me/FreeUPSCMaterials

https://telegram.me/MaterialforExam

latitudes. The Labrador Ocean current is cold current while the Gulf Stream is a warm current. The ocean current influence the temperature conditions of the area. Warm currents bring about warm temperature over land surface. S4 The areas where the warm and cold currents meet provide the best fishing grounds of the world because they help the nutrient rich deeper ocean water to churn up to the surface leading to the growth of phytoplankton which is a source of food for fishes. Q Source: Chapter 5: 7th NCERT Geography 50 The continental shelf is the extended margin of each continent occupied by relatively shallow seas and gulfs. You are most likely to encounter a long shelve in which of the following regions? A. B. C. D.

Siberian Shelf in Arctic Ocean Coasts of Chile Gulf of Kutch West Coast of Sumatra

Correct Answer : A Answer Justification : Justification: The width of the continental shelves vary from one ocean to another. The average width of continental shelves is about 80 km. The shelves are almost absent or very narrow along some of the margins like the coasts of Chile, the west coast of Sumatra, etc. On the contrary, the Siberian shelf in the Arctic Ocean, the largest in the world, stretches to 1,500 km in width. The depth of the shelves also varies. It may be as shallow as 30 m in some areas while in some areas it is as deep as 600 m. Q Source: Chapter 13: 11th NCERT Fundamentals of Physical Geography 51 Consider the following statements about the importance of arctic and sub-arctic regions. 1. Majority of world’s wetlands are located in the Arctic and sub-Arctic. 2. Majority of world’s global marine fisheries catch by weight comes from Arctic and surrounding seas. Which of the above is/are correct? A. 1 only B. 2 only C. Both 1 and 2 D. None orrect Answer : A

Answer Justification : 43

Mock Test 10 https://telegram.me/UPSCMaterials

https://telegram.me/FreeUPSCMaterials

https://telegram.me/MaterialforExam

Justification: Statement 1: Some may wonder how one can talk of abundant wetland when the Arctic is, in fact, a desert. A desert is defined by the amount of precipitation (rainfall or snowfall), and the Arctic gets very little of that. But, the water in the wetlands comes from melting ice and snow. Permafrost plays an important part in the formation of almost all Arctic wetlands and lakes. It remains under the wetlands at shallow depths and creates a barrier of frozen earth that prevents the water from draining away. There are five basic types of Arctic wetland - bogs, fens, swamps, marshes, and shallow open water. Bogs and fens are the most common. Statement 2: They produce more than 10% of global marine fisheries catches by weight The Arctic and surrounding seas also account for 5.3% of the world's crustean catch by weight. Learning: A major arctic species, Polar bears, are reported to be affected by decreasing sea ice. The global population of polar bears is predicted to decrease by 30% in the next 45 years. The ivory gull, a high Arctic species has decreased by 80% (since the 1980s) on its Canadian range. Of the 21 Northern languages that have gone extinct, 48% of those have disappeared since 1990. This indicates an increasing rate of language extinction. Q Source: Improvisation: Qs asked on Arctic: Past year UPSC papers 52 Factors affecting ocean salinity include 1. Freezing and thawing of ice in Polar Regions 2. Wind flow over the ocean 3. Areal extent of the Ocean Select the correct answer using the codes below. A. 1 and 2 only B. 2 only C. 1 and 3 only D. 1, 2 and 3

Correct Answer : D 44

Mock Test 10 https://telegram.me/UPSCMaterials

https://telegram.me/FreeUPSCMaterials

https://telegram.me/MaterialforExam

Answer Justification : Justification: Statement 1: The salinity of water in the surface layer of oceans depend mainly on evaporation and precipitation. Surface salinity is greatly influenced in coastal regions by the fresh water flow from rivers, and in polar regions by the processes of freezing and thawing of ice. Also, Baltic Sea records low salinity due to influx of river waters in large quantity. Statement 2: Wind, also influences salinity of an area by transferring water to other areas. The ocean currents contribute to the salinity variations. Salinity, temperature and density of water are interrelated. Hence, any change in the temperature or density influences the salinity of an area. The North Sea, in spite of its location in higher latitudes, records higher salinity due to more saline water brought by the North Atlantic Drift. Statement 3: The salinity variation in the Pacific Ocean is mainly due to its shape and larger areal extent. Salinity decreases on the western parts of the northern hemisphere because of the influx of melted water from the Arctic region. th

Q Source: Chapter 13: 11 NCERT Fundamentals of Physical Geography 53 Which of these processes provides salts to the oceans? 1. Evaporation 2. Submarine Volcanism 3. River inflow 4. Seepage of ocean water to and fro hydrothermal vents Select the correct answer using the codes below. A. 1 and 3 only B. 2, 3 and 4 only C. 1 and 4 only D. 1, 2, 3 and 4

Correct Answer : B Answer Justification : Justification: Statement 1: This would not add to the amount of salt in the ocean rather increase its concentration by reducing the amount of ocean water by evaporation. Statement 3: As rainwater passes through soil and percolates through rocks, it dissolves some of the minerals, a process called weathering. This is the water we drink, and of course, we cannot taste the salt because its concentration is too low. Eventually, this water with its small load of dissolved minerals or salts reaches a stream and flows into lakes and the ocean. The annual addition of dissolved salts by rivers is only a tiny fraction of the total salt in the ocean. The dissolved salts 45

Mock Test 10 https://telegram.me/UPSCMaterials

https://telegram.me/FreeUPSCMaterials

https://telegram.me/MaterialforExam

carried by all the world’s rivers would equal the salt in the ocean in about 200 to 300 million years. Statement 2: Another process that provides salts to the oceans is submarine volcanism, the eruption of volcanoes under water. This is similar to the previous process in that seawater is reacting with hot rock and dissolving some of the mineral constituents. Statement 4: Rivers are not the only source of dissolved salts. Found on the crest of oceanic ridges hydrothermal vents, represent places on the ocean floor where sea water that has seeped into the rocks of the oceanic crust, has become hotter, and has dissolved some of the minerals from the crust, now flows back into the ocean. With the hot water comes a large complement of dissolved minerals. Estimates of the amount of hydrothermal fluids now flowing from these vents indicate that the entire volume of the oceans could seep through the oceanic crust in about 10 million years. Thus, this process has a very important effect on salinity. The reactions between seawater and oceanic basalt, the rock of ocean crust, are not one-way, however; some of the dissolved salts react with the rock and are removed from the water. th

Q Source: Chapter 13: 11 NCERT Fundamentals of Physical Geography 54 The government has announced the Drone Regulations 1.0. These regulations will enable the safe, commercial usage of drones starting December 1, 2018. Consider the following about them. 1. They are intended to enable visual line-of-sight daytime-only and a maximum of 1000 ft altitude operations. 2. All such drones, with some exceptions, are to be registered and issued with Unique Identification Number (UIN). 3. The regulation defines some “No Drone Zones”, inter alia, around airports and near international borders. 4. Operations of these Remotely Piloted Aircraft System (RPAS) cannot be enabled through Digital Sky Platform. Select the correct answer using the codes below. A. 1, 2 and 3 only B. 2 and 3 only C. 1, 3 and 4 only D. 1, 2 and 4 only

Correct Answer : B Answer Justification : Justification: As per the regulation, there are 5 categories of RPAS categorized by weight, namely nano, micro, small, medium and large. Operational/ Procedural Requirements:

46

Mock Test 10 https://telegram.me/UPSCMaterials

https://telegram.me/FreeUPSCMaterials

https://telegram.me/MaterialforExam

All RPAS except nano and those owned by NTRO, ARC and Central Intelligence Agencies are to be registered and issued with Unique Identification Number (UIN). Unmanned Aircraft Operator Permit (UAOP) shall be required for RPA operators except for nano RPAS operating below 50 ft., micro RPAS operating below 200 ft., and those owned by NTRO, ARC and Central Intelligence Agencies. The mandatory equipment required for operation of RPAS except nano category are (a) GNSS (GPS), (b) Return-To-Home (RTH), (c) Anti-collision light, (d) ID-Plate, (e) Flight controller with flight data logging capability, and (f) RF ID and SIM/ No-Permission No Take off (NPNT). For flying in controlled Airspace, filing of flight plan and obtaining Air Defence Clearance (ADC) /Flight Information Centre (FIC) number shall be necessary. Minimum manufacturing standards and training requirements of Remote Pilots of small and above categories of RPAS have been specified in the regulation. Other highlights: No Drone Zones: The regulation defines “No Drone Zones” around airports;near international border, Vijay Chowk in Delhi; State Secretariat Complex in State Capitals, strategic locations/vital and military installations; etc. Statement 4: Operations through Digital Platform: Operations of Remotely Piloted Aircraft System (RPAS) to be enabled through Digital Sky Platform. The RPAS operations will be based on NPNT (No Permission, No Take off). There will be different colour zones visible to the applicant while applying in the digital sky platform, viz, Red Zone: flying not permitted, Yellow Zone (controlled airspace): permission required before flying, andGreen Zone (uncontrolled airspace): automatic permission. Q Source: As mentioned above 55 Changes in which of the following wave parameters may lead to change in the Wave period? 1. Wave frequency 2. Wavelength 3. Wave height 4. Wave speed Select the correct answer using the codes below. A. 1, 2 and 3 only B. 1, 2 and 4 only C. 1 and 4 only D. 2 and 3 only orrect Answer : B

Answer Justification : 47

Mock Test 10 https://telegram.me/UPSCMaterials

https://telegram.me/FreeUPSCMaterials

https://telegram.me/MaterialforExam

Justification: Statement 1: Since wave frequency is the number of waves per second, and the period is essentially the number of seconds per wave, the relationship between frequency and period is

just as in the case of harmonic motion of an object. We can see from this relationship that a higher frequency means a shorter period. Statement 3: The height of the crest of a wave does not determine its frequency or speed or period. Statement 3 and 4: The speed of propagation vw is the distance the wave travels in a given time, which is one wavelength in a time of one period. In equation form, it is written as

This implies that wavelength, time period, frequency and wave speed are interrelated. Q Source: Chapter 14: 11th NCERT Fundamentals of Physical Geography 56 Qubit is A. B. C. D.

The basic unit of quantum information A subatomic part of quarks and gluons The sun facing side of a geo-synchronous satellite The theory that links classical and sub-atomic mechanics

Correct Answer : A Answer Justification : Learning: In quantum computing, or quantum bit (sometimes qbit) is the basic unit of quantum information — the quantum version of the classical binary bit physically realized with a two-state device. A qubit is a two-state (or two-level) quantum-mechanical system, one of the simplest quantum systems displaying the weirdness of quantum mechanics. 48

Mock Test 10 https://telegram.me/UPSCMaterials

https://telegram.me/FreeUPSCMaterials

https://telegram.me/MaterialforExam

Examples include: the spin of the electron in which the two levels can be taken as spin up and spin down; or the polarization of a single photon in which the two states can be taken to be the vertical polarization and the horizontal polarization. In a classical system, a bit would have to be in one state or the other. However, quantum mechanics allows the qubit to be in a coherent superposition of both states/levels at the same time, a property that is fundamental to quantum mechanics and thus quantum computing. Q Source: Surprise questions 57 The flow of Ocean currents will be significantly affected if 1. The shape of earth changes leading to changes in gravitational forces 2. Earth starts rotating at a different velocity on its axis Which of the above is/are correct? A. 1 only B. 2 only C. Both 1 and 2 D. None

Correct Answer : C Answer Justification : Justification: Statement 1: Gravity tends to pull the water down to pile and create gradient variation. Gravity depends on the radius or shape of the earth and its mass. If any of this will change the gravitational force will change. Statement 2: The Coriolis force intervenes and causes the water to move to the right in the northern hemisphere and to the left in the southern hemisphere. Coriolis force depends on the rotation velocity and shape of the earth. You can see now why S2 is correct. Background: Ocean currents are like river flow in oceans. They represent a regular volume of water in a definite path and direction. Ocean currents are influenced by two types of forces namely : (i) primary forces that initiate the movement of water; (ii) secondary forces that influence the currents to flow. The primary forces that influence the currents are: (i) heating by solar energy; (ii) wind; (iii) gravity; (iv) coriolis force. Heating by solar energy causes the water to expand. That is why, near the equator the ocean water is about 8 cm higher in level than in the middle latitudes. This causes a very slight gradient and water tends to flow down the slope. Wind blowing on the surface of the ocean pushes the water to move. Friction between the wind and the water surface affects the movement of the water body in its course. 49

Mock Test 10 https://telegram.me/UPSCMaterials

https://telegram.me/FreeUPSCMaterials

https://telegram.me/MaterialforExam

th

Q Source: Chapter 14: 11 NCERT Fundamentals of Physical Geography 58 Consider the following statements. 1. The North Equatorial Current is separated from the equatorial circulation by the Equatorial Countercurrent. 2. In the Indian Ocean, the westward-flowing South Equatorial Current is well-developed only slightly north of the equator. Which of the above is/are correct? A. 1 only B. 2 only C. Both 1 and 2 D. None orrect Answer : A

Answer Justification : Justification: Statement 1: The North Equatorial Current is a significant Pacific and Atlantic Ocean current that flows east-to-west between about 10° north and 20° north. It is the southern side of a clockwise subtropical gyre. Despite its name, the North Equatorial Current is not connected to the equator. In both oceans, it is separated from the equatorial circulation by the Equatorial Countercurrent (also known as the North Equatorial Countercurrent), which flows eastward. The westward surface flow at the equator in both oceans is part of the South Equatorial Current. Statement 2: South Equatorial Current is the current in the Pacific, Atlantic, and Indian Ocean that flows east-to-west between the equator and about 20 degrees south. In the Pacific and Atlantic Oceans, it extends across the equator to about 5 degrees north. In the Indian Ocean, the westward-flowing South Equatorial Current is well-developed only south of the equator. Directly on the equator, the winds reverse twice a year due to the monsoons, and so the surface current can be either eastward or westward. Within the southern hemisphere, the South Equatorial Current is the westward limb of the very large-scale subtropical gyres. These gyres are driven by the combination of trade winds in the tropics and westerly winds that are found south of about 30 degrees south, through a rather complicated process that includes western boundary current intensification. On the equator, the South Equatorial Current is driven directly by the trade winds which blow from east to west.

50

Mock Test 10 https://telegram.me/UPSCMaterials

https://telegram.me/FreeUPSCMaterials

https://telegram.me/MaterialforExam

Q Source: Additional Research on Figure 14.3 from Chapter 14: 11th NCERT Fundamentals of Physical Geography 59 Confirmed valuable minerals found on Moon include 1. Titanium 2. Helium-3 3. Manganese (Mn) Select the correct answer using the codes below. A. 1 only B. 1 and 2 only C. 1 and 3 only D. 2 and 3 only

Correct Answer : C Answer Justification : Justification: Elements known to be present on the lunar surface include, among others, oxygen (O), silicon (Si), iron (Fe), magnesium (Mg), calcium (Ca), aluminium (Al), manganese (Mn) and titanium (Ti). Among the more abundant are oxygen, iron and silicon. The oxygen content is estimated at 45% (by weight). Carbon (C) and nitrogen (N) appear to be present only in trace quantities from deposition by solar wind. Statement 1: The titanium-rich areas on the moon puzzled the researchers. The highest abundance of titanium in similar rocks on Earth hovers around 1 percent or less, the scientists explained. The new map shows that these troves of titanium on the moon range from about 1 percent to a little more than 10 percent. Titanium on the moon is primarily found in the mineral ilmenite, a compound that contains iron, titanium and oxygen. If humans one day mine on the moon, they could break down ilmenite to separate these elements. 51

Mock Test 10 https://telegram.me/UPSCMaterials

https://telegram.me/FreeUPSCMaterials

https://telegram.me/MaterialforExam

Statement 2: Helium-3 is predicted to be present on the Moon; hasn’t been confirmed yet. It is a component of the solar wind, which is constantly blowing outward from the sun. Our atmosphere blocks helium-3 from reaching Earth’s surface. The moon doesn’t have much atmosphere, so scientists speculate that there may be more than a million tons of the isotope up there. The dominant theory is that a Mars-size object struck Earth 4.5 billion years ago, breaking off a bunch of material that melded together to form the moon. If this theory is correct, we can expect that the moon is made of roughly the same chemical building blocks as Earth. Q Source: https://www.space.com/13247-moon-map-lunar-titanium.html 60 Consider the following statements about the Geography of continents. 1. Asia is separated from Europe by the Alps Mountains on the west. 2. The Arctic Circle passes through Europe. 3. North America is linked to South America by a narrow strait of land. Select the correct answer using the codes below. A. 1 and 2 only B. 2 and 3 only C. 1 and 3 only D. 1, 2 and 3

Correct Answer : B Answer Justification : Justification: Statement 1: The diagram below shows it. It is Ural mountains.

Statement 2:

52

Mock Test 10 https://telegram.me/UPSCMaterials

https://telegram.me/FreeUPSCMaterials

https://telegram.me/MaterialforExam

Statement 3: It is the Isthmus of Panama that connects North with South America. th

Q Source: Page 32-33: NCERT 6 Geography: The Earth Our Habitat 61 The “capabilities approach” of Amartya Sen brings a paradigm shift in the way poverty is being looked at by policymakers. Suppose if “not having a resource” is generally termed as poverty, then which of the following positions will be taken by this approach? 1. A person who has this resource but cannot use it will still be called as poor. 2. A person who does not have this resource will not be termed poor because not having a resource does not make the person necessarily “unhappy”. Which of the above arguments is\are correct in this context? A. 1 only B. 2 only C. Both 1 and 2 D. None

Correct Answer : A Answer Justification : Justification & Learning: His approach mainly relies on two major aspects: 1) Functionalities and 2) Capabilities. Functionality means having an instrument or value. Examples of functionings can vary from 53

Mock Test 10 https://telegram.me/UPSCMaterials

https://telegram.me/FreeUPSCMaterials

https://telegram.me/MaterialforExam

elementary things, such as being healthy, having a good job, and being safe, to more complex states, such as being happy, having self-respect, and being calm. In this question, functionality means having a cycle. So, those who have cycles will not be poor. Statement 2 is incorrect. Capabilities are the functionality that a person can achieve or perform. For e.g. in this question, he has a cycle, but if he can’t ride it due to disability (incapability), he is capability poor, and thus will be declared a poor. Statement 1 is correct. Seen from another perspective, For example, the difference between fasting and starving, on person's well-being, is whether the person is choosing not to eat. In this example, the functioning is starving but the capability to obtain an adequate amount of food is the key element in evaluating well-being between individuals in the two states. In sum, having a lifestyle is not the same as choosing it; well-being depends on how that lifestyle came to be. Q Source: Additional Research: UPSC Prelims 2016 question on growth 62 LIFE- MGNREGA is a restructured version of MGNREGA. The new scheme focuses on 1. Skill development of MGREGA workers 2. Devolution of taxation powers to PRIs to fund MGREGA projects 3. Absorption of MGNREGA workers in the farming sector Select the correct answer using the codes below. A. 1 and 2 only B. 2 and 3 only C. 1 only D. 1 and 3 only orrect Answer : C

Answer Justification : Justification: It is also known as Livelihoods in Full Employment (LIFE) Project – MGNREGA. With an aim to promote self reliance and improve the skill base of MGNREGA workers, the government has formulated the new project linking MGNREGA to skill development programmes. The project has been formulated in consultation with Deen Dayal Upadhaya Grameen Kaushalya Yojna (DDU-GKY) and National Rural Livelihood Mission (NRLM). The states have been asked to prepare the plans.

54

Mock Test 10 https://telegram.me/UPSCMaterials

https://telegram.me/FreeUPSCMaterials

https://telegram.me/MaterialforExam

Q Source: Additional Research: UPSC Prelims questions on MGNREGA asked few times 63 Consider the following about the South Asian Association for Regional Cooperation (SAARC) Summit. 1. It was founded along with the Non-alignment movement (NAM). 2. SAARC Secretariat is based in New Delhi to facilitate effective cooperation between members. 3. Sri Lanka and Pakistan joined SAARC after 2008 Global recession. 4. Afghanistan is not a member of SAARC Summit. Select the correct answer using the codes below. A. 1 and 2 only B. 2 and 4 only C. 3 and 4 only D. None of the above

Correct Answer : D Answer Justification : Justification: Statement 1: The idea for the SAARC was proposed by Ziaur Rahman, the then President of Bangladesh on May 2, 1980. The seven founding countries had met for first time in April 1981 and then in 1985, they created SAARC Charter. NAM belongs to the 1960s and thus 1 would be wrong. Statement 2: The first SAARC summit was held in Dhaka (Bangladesh) in December 1985. The SAARC Secretariat was established in Kathmandu (Nepal) in 1987. Statement 3: They are founding member states. Afghanistan joined late in 2007. So, 3 and 4 both are incorrect. Learning: Members are: Bangladesh, Bhutan, India, Maldives, Nepal, Pakistan, Sri Lanka and Afghanistan Observer states are Australia, China, European Union, Japan, Iran, Mauritius, Myanmar, South Korea, and United States. Q Source: http://indianexpress.com/article/india/india-news-india/home-minister-rajnath-singh-to-visit-pakistan -despite-threat-by-hafiz-saeed-2948437/ 64 The Reserve Bank of India (RBI) had introduced an incremental Cash Reserve Ratio (CRR) few years ago to 1. Allow banks to cut down their priority sector lending targets 2. Absorb excess liquidity in the banking system following demonetisation 3. Increase the loanable funds with RBI Select the correct answer using the codes below. 55

Mock Test 10 https://telegram.me/UPSCMaterials

A. B. C. D.

https://telegram.me/FreeUPSCMaterials

https://telegram.me/MaterialforExam

1 and 2 only 2 and 3 only 2 only 1 and 3 only

Correct Answer : C Answer Justification : Justification: Statement 1: PSL are administered independently of the CRR requirements. Banks must lend a portion of their deposits in form of PSL. The recent move was not related to PSL. Statement 2: RR is the proportion of deposits that banks have to keep as cash with the RBI. The present requirement is only 4% of total demand deposits. But, as per the recent measure Banks have to maintain 100% CRR for incremental deposits they received between September 16, 2016 and November 11, 2016. This is because over 2 lakh crores of cash was deposited in the banking system. Lending this fund will create a massive rise in liquidity in the financial markets and lead to inflation and volatility. Hence, these funds are not allowed to be lent as of now.

Q Source: Additional Research: UPSC Prelims questions on CRR 65 On the south-western coasts of India, you are most likely to come across which of these forests? A. B. C. D.

Tropical deciduous forests Mangrove forests Tropical rainforests Montane forests

Correct Answer : C Answer Justification : Learning: These forests are restricted to heavy rainfall areas of the Western Ghats (near southwestern coasts) and the island groups of Lakshadweep, Andaman and Nicobar, upper parts of Assam and Tamil Nadu coast. They are at their best in areas having more than 200 cm of rainfall with a short dry season. The trees reach great heights. Since the region is warm and wet throughout the year, it has a luxuriant vegetation of all kinds – trees, shrubs, and creepers giving it a multilayered structure. There is no definite time for trees to shed their leaves. As such, these forests appear green all the year round.

56

Mock Test 10 https://telegram.me/UPSCMaterials

https://telegram.me/FreeUPSCMaterials

https://telegram.me/MaterialforExam

Q Source: NCERT 9th Geography 66 Voting shares in the United Nations General Assembly are distributed between members on the basis of A. B. C. D.

Population Economic Size Contribution to UN All members have equal vote

57

Mock Test 10 https://telegram.me/UPSCMaterials

https://telegram.me/FreeUPSCMaterials

https://telegram.me/MaterialforExam

Correct Answer : D Answer Justification : Learning: Voting in the General Assembly on important questions, namely, recommendations on peace and security, budgetary concerns and the election, admission, suspension or expulsion of members – is by a two-thirds majority of those present and voting. Other questions are decided by a straightforward majority. Each member country has one vote. Apart from approval of budgetary matters, including adoption of a scale of assessment, Assembly resolutions are not binding on the members. The Assembly may make recommendations on any matters within the scope of the UN, except matters of peace and security under Security Council consideration. The one state, one vote power structure potentially allows states comprising just five percent of the world population to pass a resolution by a two-thirds vote. Q Source: Page 15: Democratic Politics - I 67 Minerals on earth mainly come from A. B. C. D.

Magma Fossils Meteoroids Both (a) and (b)

Correct Answer : D Answer Justification : Learning: Magma cools slowly as it rises towards Earth’s surface. It can take thousands to millions of years to become solid when it is trapped inside Earth. As the magma cools, solid rocks form (igneous). These rocks are mixtures of minerals. Granite is a common rock that forms when magma cools. Granite contains the minerals quartz, plagioclase feldspar, and potassium feldspar. The same igneous rock can be eroded, weathered, transported, deposited and consolidated as sedimentary rocks which are the source of non-metallic minerals.

58

Mock Test 10 https://telegram.me/UPSCMaterials

https://telegram.me/FreeUPSCMaterials

https://telegram.me/MaterialforExam

Moreover, fossils are formed by decomposition of organic material in situ. After long time they get converted in useful hydrocarbons like petroleum and coal. Q Source: Fundamentals of Physical Geography: 11th NCERT 68 Sinai Peninsula lies between A. B. C. D.

Gulf of Suez and Gulf of Aqaba Gulf of Aden and Gulf of Aqaba Gulf of Suez and Gulf of Aden Gulf of Masirah and Gulf of Aqaba

Correct Answer : A Answer Justification : Learning: It is situated between the Mediterranean Sea to the north and the Red Sea to the south, serving as a land bridge between Asia and Africa. It is the only part of Egyptian territory located in Asia. Israel invaded and occupied Sinai during the Suez Crisis. As a result of the Israel-Egypt Peace Treaty of 1979 and subsequent efforts, Israel withdrew from all of the Sinai Peninsula.

Q Source: Map based question: Region: West Asia

59

Mock Test 10 https://telegram.me/UPSCMaterials

https://telegram.me/FreeUPSCMaterials

https://telegram.me/MaterialforExam

69 Consider the following statements. 1. Photon assists electromagnetic radiation. 2. Electron-neutrinos are produced in large numbers during supernova explosions. Which of the above is/are correct? A. 1 only B. 2 only C. Both 1 and 2 D. None

Correct Answer : C Answer Justification : Justification: Statement 1: There are two types of fundamental particles: matter particles, some of which combine to produce the world about us, and force particles – one of which, the photon, is responsible for electromagnetic radiation. These are classified in the standard model of particle physics, which theorises how the basic building blocks of matter interact, governed by fundamental forces. Matter particles are fermions while force particles are bosons. Statement 2: The charged electron is responsible for electric currents. Its uncharged partner, known as the electron-neutrino, is produced copiously in the sun and these interact so weakly with their surroundings that they pass unhindered through the Earth. A million of them pass through every square centimetre of your body every second, day and night. Electron-neutrinos are produced in unimaginable numbers during supernova explosions and it is these particles that disperse elements produced by nuclear burning into the universe. These elements include the carbon from which we are made, the oxygen we breathe, and almost everything else on earth. Therefore, in spite of the reluctance of neutrinos to interact with other fundamental particles, they are vital for our existence. Q Source: Additional Research: http://www.insightsonindia.com/2018/08/29/insights-daily-current-affairs-29-august-2018/ 70 A group of experts, constituted by NITI Aayog, has urged the government to set up a dedicated mission to salvage and revive spring water systems in the country’s Himalayan States based on the report “Inventory and Revival of Springs in the Himalayas for Water Security, 2018”. Consider the following with reference to it. 1. Almost half of the perennial springs in the region have already dried up or have become seasonal. 2. Majority of the cultivable area in the Himalayas is fed by natural springs. 3. Sikkim has the greatest density of natural springs in Eastern Himalayas. Select the correct answer using the codes below. A. 1, 2 and 3 B. 2 and 3 only C. 1 and 2 only D. 1 and 3 only 60

Mock Test 10 https://telegram.me/UPSCMaterials

https://telegram.me/FreeUPSCMaterials

https://telegram.me/MaterialforExam

Correct Answer : A Answer Justification : Justification: Almost half of the perennial springs have already dried up or have become seasonal and tens of thousands of villages are currently facing acute water shortage for drinking and other domestic purposes,” the group noted in its report titled ‘Inventory and Revival of Springs in the Himalayas for Water Security.’ “Almost 60% of low-discharge springs that provided water to small habitations in the Himalayan region have reported clear decline during the last couple of decades with almost 64% of the cultivable area in the Himalayas fed by natural springs, they are often the only source of irrigation in the region. The report noted that there were also multiple sources of pollution in springs and these were due to both geogenic, or ‘natural’ causes and anthropogenic, or man-made, ones. Microbial content, sulphates and nitrates were primarily because of anthropogenic reasons and contamination from fluoride, arsenic and iron was mainly derived from geogenic sources. Coliform bacteria in spring water could originate from septic tanks, household wastewater, livestock facilities, and manure lagoons in the source area or in the aquifers feeding springs. Similarly, nitrate sources were septic tanks, household wastewater, agricultural fertilisers, and livestock facilities. While Meghalaya with 3,810 villages with springs had the highest number of these water sources in the Eastern Himalayan States, Sikkim had the greatest density with 94% of its villages having a spring. In the Western Himalayas, Jammu & Kashmir had both the highest number of villages with springs at 3,313 and the greatest density of 50.6%. The task force moots an 8-year programme to overhaul spring water management. This includes: preparing a digital atlas of the country’s springsheds, training ‘para-hydrogeologists’ who could lead grassroots conservation and introduction of a ‘Spring Health Card.’ Q Source: https://www.thehindu.com/news/national/other-states/panel-urges-plan-to-save-himalayan-springs/ar ticle24857637.ece 71 Biological control of pests and plant diseases is helpful because 1. Chemical methods of pest control indiscriminately kill both useful and harmful pests and insects. 2. Greater biodiversity at the farm increases the chances of plant survival and further plant growth. Which of the above is/are correct? A. 1 only B. 2 only C. Both 1 and 2 D. None

61

Mock Test 10 https://telegram.me/UPSCMaterials

https://telegram.me/FreeUPSCMaterials

https://telegram.me/MaterialforExam

Correct Answer : C Answer Justification : Justification: Statement 1 and 2: A key belief of the organic farmer is that biodiversity furthers health. The more variety a landscape has, the more sustainable it is. So, 1 is correct. The organic farmer, therefore, works to create a system where the insects that are sometimes called pests are not eradicated, but instead are kept at manageable levels by a complex system of checks and balances within a living and vibrant ecosystem. The organic farmer holds the view that the eradication of the creatures that are often described as pests is not only possible, but also undesirable, for without them the beneficial predatory and parasitic insects which depend upon them as food or hosts would not be able to survive. Q Source: Improvisation: Qs asked on Organic Farming: Past year UPSC papers 72 Cycle of ‘Solar spots’ can significantly affect the climate on earth by leading to a 1. Change in solar output 2. Reversal of earth’s magnetic field Which of the above is/are correct? A. 1 only B. 2 only C. Both 1 and 2 D. None

Correct Answer : A Answer Justification : Justification: Statement 1: Sunspots are temporary phenomena on the photosphere of the Sun that appear visibly as dark spots compared to surrounding regions. For instance, solar energetic particles and cosmic rays could reduce ozone levels in the stratosphere. This in turn alters the behavior of the atmosphere below it, perhaps even pushing storms on the surface off course They reduce solar output if increase in number. So, 1 is correct. Statement 2: The Sun's magnetic field structures its atmosphere and outer layers all the way through the corona and into the solar wind. Solar sports reflect intense magnetic activity on the Sun at those spots, and not earth. Q Source: http://www.insightsonindia.com/2018/06/27/insights-daily-current-affairs-27-june-2018/

62

Mock Test 10 https://telegram.me/UPSCMaterials

https://telegram.me/FreeUPSCMaterials

https://telegram.me/MaterialforExam

73 Which of the following are important limitations of Human Development Index (HDI)? #000000 1. It does not contain social exclusion and discrimination. 2. It ignores the issue of political freedom. 3. It does not take into account income inequality. Select the correct answer using the codes below. A. 1 and 3 only B. 2 only C. 1 and 2 only D. 1, 2 and 3

Correct Answer : D Answer Justification : Justification: HDI covers three major indicators – health, standard of living (per capita income) and educational attainments. In calculating the above, it takes only gross national figures into account. It does not capture qualitative aspects of indicators. For e.g. one year additional schooling in rural India and urban India are significantly different. Also, an extra year lived at the age of 81 might not given an individual the same utility as that lived at the age 25. In several countries, primary enrolment does not mean completion of education. Drop out rates are not counted. It does not contain social exclusion, discrimination, social inequity, gender disparities I development. So, higher HDI does not necessarily ensure greater happiness and cultural satisfaction. Q Source: Improvisation: Qs asked on HDI: Past year UPSC papers 74 Mongmong is a major festival of the Sangtams, who are the largest of the three principal tribes A. B. C. D.

Nagaland Sikkim Mizoram Manipur

Correct Answer : A Answer Justification : Learning: They are situated in Kiphire district bordering Myanmar. The festival is celebrated from September 1-6 every year as a festival of togetherness, forgiveness and prayer for a bountiful harvest. 63

Mock Test 10 https://telegram.me/UPSCMaterials

https://telegram.me/FreeUPSCMaterials

https://telegram.me/MaterialforExam

The Sangtams have about 12 festivals spread all over the¿ Calendar Year with feasting, dance, and music built around the community life of the people. Except certain gennas, all the festivals are concerned with food production, blessings and prosperity. Mongmong is one of the most important festivals of the Sangtams. The predominant themes of the festivals were mostly religious and spiritual rituals where people predominantly offer religious and spiritual rituals to the Supreme Being. The festival is observed in the first week of September every year. Q Source: https://www.thehindu.com/news/national/other-states/relief-rice-sweetens-festival-in-nagaland-distri ct/article24857967.ece 75 Consider the following about Congress socialist party. 1. It was formed within the Congress immediately after the failure of the Non-cooperation movement. 2. It rejected the ideology of democracy and supported a Maoist communist revolution. 3. It later restructured itself as the All India Kisan Sabha (AIKS). Select the correct answer using the codes below. A. 1 only B. 2 and 3 only C. 3 only D. None of the above

Correct Answer : D Answer Justification : Justification: Statement 1: The origins of the Socialist Party can be traced back to the mass movement stage of the Indian National Congress in the pre-independence era. The Congress Socialist Party (CSP) was formed within the Congress in 1934 by a group of young leaders who wanted a more radical and egalitarian Congress. In 1948, the Congress amended its constitution to prevent its members from having a dual party membership. This forced the Socialists to form a separate Socialist Party in 1948. Statement 2: The socialists believed in the ideology of democratic socialism which distinguished them both from the Congress as well as from the Communists. They criticised the Congress for favouring capitalists and landlords and for ignoring the workers and the peasants. Statement 3: But the socialists faced a dilemma when in 1955 the Congress declared its goal to be the socialist pattern of society. Thus it became difficult for the socialists to present themselves as an effective alternative to the Congress. Some of them, led by Rammanohar Lohia, increased their distance from and criticism of the Congress party. Some others like Asoka Mehta advocated a limited cooperation with the Congress.

64

Mock Test 10 https://telegram.me/UPSCMaterials

https://telegram.me/FreeUPSCMaterials

https://telegram.me/MaterialforExam

The Socialist Party went through through many splits and reunions leading to the formation of many socialist parties. These included the Kisan Mazdoor Praja Party, the Praja Socialist Party and Samyukta Socialist Party. Jayaprakash Narayan, Achyut Patwardhan, Asoka Mehta, Acharya Narendra Dev, Rammanohar Lohia and S.M. Joshi were among the leaders of the socialist parties. Many parties in contemporary India, like the Samajwadi Party, the Rashtriya Janata Dal, Janata Dal (United) and the Janata Dal (Secular) trace their origins to the Socialist Party. th

Q Source: Revision 12 NCERT: Chapter on Congress and other political parties: Politics after Independence 76 Which of these rights is enjoyed by a Non-Resident Indian (NRI), but NOT by an Overseas Citizen of India (OCI)? A. B. C. D.

Right to vote Right to dual citizenship Right to contest for public office Both (a) and (c)

Correct Answer : D Answer Justification : Justification: NRI is a citizen of India who holds an Indian passport and has temporarily emigrated to another country for six months or more for employment, residence, education or any other purpose. A Overseas Citizen of India (OCI) is a PIO who had migrated to other countries and acquired citizenship (except Afghanistan and Bangladesh) there. If their country of residence allows dual citizenship they are eligible to hold a OCI card. Overseas Citizenship of India is not an actual citizenship of India and thus, does not amount to dual citizenship or dual nationality. An Overseas Citizen of India will enjoy all rights and privileges available to Non-Resident Indians on a parity basis excluding the right to invest in agriculture and plantation properties or hold public office, and the right to vote. Learning: A Person of Indian Origin (PIO) is a person of Indian origin or ancestry but who is not a citizen of India and is the citizen of another country (except South Asian countries). A PIO might have been a citizen of India and subsequently taken the citizenship of another country, or have ancestors born in India or other states.

65

Mock Test 10 https://telegram.me/UPSCMaterials

https://telegram.me/FreeUPSCMaterials

https://telegram.me/MaterialforExam

Q Source: Revision of Chapter on Citizenship: Indian Polity: M Laxmikanth 77 Chinook is an important local wind prevalent in the rocky mountain slopes of USA. Similarly, match the following local winds with the area of their prevalence. 1. Mistral : North African desert 2. Foehn : Southern slopes of Alps 3. Sirocco : Appalachian mountains Select the correct matches using the codes below. A. 1 and 3 only B. 1 and 2 only C. 2 only D. None of the above

Correct Answer : D Answer Justification : Justification: Statement 1: It flows in Southern slopes of Alps. It is a strong, cold, northwesterly wind that blows from southern France into the Gulf of Lion in the northern Mediterranean, with sustained high speed winds. Statement 2: It flows in Northern slopes of Alps. It is a rain shadow wind that results from the subsequent adiabatic warming of air that has dropped most of its moisture on windward slopes. Statement 3: It flows in African Deserts. It is a Mediterranean wind that comes from the Sahara and can reach hurricane speeds in North Africa and Southern Europe. Q Source: Improvisation: Past year UPSC papers 78 Consider the following statements. Department of Official Language 1. It is a department under Ministry of Home Affairs. 2. It has the nodal responsibility for all matters relating to the progressive use of Hindi as the Official Language of the Union. 3. Its functions are determined in accordance with the Government of India (Allocation of Business) Rules, 1961. Select the correct answer using the codes below. A. 1 and 2 only B. 2 and 3 only C. 1 and 3 only D. 1, 2 and 3

Correct Answer : D

66

Mock Test 10 https://telegram.me/UPSCMaterials

https://telegram.me/FreeUPSCMaterials

https://telegram.me/MaterialforExam

Answer Justification : Justification: With a view to ensuring compliance of the constitutional and legal provisions regarding official language and to promote the use of Hindi for the official purposes of the Union, the Department of Official Language was set up in June 1975 as an independent Department of the Ministry of Home Affairs. Since then, this Department has been making efforts for accelerating the progressive use of Hindi for the official purposes of the Union. In accordance with the Government of India (Allocation of Business) Rules, 1961, this Department has been entrusted with the following items of work: Implementing the provisions of the Constitution relating to the Official Language and the provisions of the Official Languages Act, 1963 (19 of 1963), except to the extent such implementation has been assigned to any other Department. Prior approval of the President for authorising the limited use of a language, other than English, in the proceedings in the High Court of a State. Nodal responsibility for all matters relating to the progressive use of Hindi as the Official Language of the Union including Hindi Teaching Scheme for Central Government Employees and publication of magazines, journals & other literature related thereto. Co-ordination in all matters relating to the progressive use of Hindi as the Official Language of the Union, including administrative terminology, syllabi, textbooks, training courses and equipment (with standardised script) required thereof. Q Source: http://www.insightsonindia.com/2018/09/01/insights-daily-current-affairs-01-september-2018/ 79 The system of ‘dyarchy’ introduced by the Government of India Act of 1919 meant that A. Both Centre and provincial legislatures had the power of legislate in their own spheres. B. Both the British Parliament and the Central Legislature had the right to make laws for India C. Certain subjects were devolved to the Indian members of the Viceroy’s executive council and certain others to the British members of the executive council D. None of the above

Correct Answer : D Answer Justification : Learning: The 1919 Act relaxed the central control over the provinces by demarcating and separating the central and provincial subjects. The central and provincial legislatures were authorised to make laws on their respective list of subjects. However, the structure of government continued to be centralised and unitary. 67

Mock Test 10 https://telegram.me/UPSCMaterials

https://telegram.me/FreeUPSCMaterials

https://telegram.me/MaterialforExam

It further divided the provincial subjects into two parts—transferred and reserved. The transferred subjects were to be administered by the governor with the aid of ministers responsible to the legislative Council. The reserved subjects, on the other hand, were to be administered by the governor and his executive council without being responsible to the legislative Council. This dual scheme of governance was known as ‘dyarchy’—a term derived from the Greek word di-arche which means double rule. However, this experiment was largely unsuccessful. Q Source: Revision of Chapter 1: Indian Polity: M Laxmikanth 80 The organization “Arctic Circle” is a/an A. B. C. D.

Intergovernmental body of rim states of Arctic Ocean A non-profit organization to facilitate dialogue between various stakeholders A body of scientists researching on the impact of climate change in arctic None of the above

Correct Answer : B Answer Justification : Learning: The Arctic Circle is the largest network of international dialogue and cooperation on the future of the Arctic. It is an open democratic platform with participation from governments, organizations, corporations, universities and others interested in the development of the Arctic and its consequences for the future of the globe. It is a non-profit and nonpartisan organization. The Arctic Circle held its first open assembly in 2013. Q Source: Improvisation: Qs asked on Arctic: Past year UPSC papers 81 Massive sedimentary deposits received over a long time by this region of the Ocean later becoming the source of fossil fuels. A. B. C. D.

Guyot Continental Slope Continental Shelf Oceanic Trench

68

Mock Test 10 https://telegram.me/UPSCMaterials

https://telegram.me/FreeUPSCMaterials

https://telegram.me/MaterialforExam

Correct Answer : C Answer Justification : Justification: Option C The continental shelves are covered with variable thicknesses of sediments brought down by rivers, glaciers, wind, from the land and distributed by waves and currents. Massive sedimentary deposits received over a long time by the continental shelves, become the source of fossil fuels. Option B The continental slope connects the continental shelf and the ocean basins. It begins where the bottom of the continental shelf sharply drops off into a steep slope. The accumulation of sediment is difficult here. Option A Guyot is a high region in the ocean like a seamount. Check the image.

th

Q Source: Chapter 13: 11 NCERT Fundamentals of Physical Geography 82 The S&D box, with reference to WTO agreements, is related to A. B. C. D.

Special exemptions for developing countries Cross border customs clearances Preferential rules of origin for LDCs Phasing out of toxic substances in exports of developed countries

Correct Answer : A Answer Justification :

69

Mock Test 10 https://telegram.me/UPSCMaterials

https://telegram.me/FreeUPSCMaterials

https://telegram.me/MaterialforExam

Learning: It comes under some of the crucial development-related issues in WTO which also include establishment of a Development Box, special and differential treatment (S&D) for developing countries, single commodity producers treatment, and the status of small island developing states (SIDS). S&D box gives the developing countries required flexibility in several WTO agreements to adapt to changed rules so as to create a level playing field with developed countries. Q Source: Revision: Test 9 Syllabus 83 Many of areas with “high ranges of tides” are located in the areas of Alaska, Canada, and northern Europe. Which of the following can be peculiar reason(s) for the same? 1. They are all located at higher latitudes. 2. The “constriction” of the oceans due to landmass distribution in the Northern hemisphere causes high tides. 3. Heavy storm waters coincide with major tidal events. Select the correct answer using the codes below. A. 1 and 3 only B. 2 only C. 1 and 2 only D. 1, 2 and 3 orrect Answer : B

Answer Justification : Justification: Statement 1: That many of the areas of the world with high ranges of tides are in the areas of Alaska, Canada, and northern Europe has created a misconception that the range of tide increases with increasing latitude (as one moves farther from the equator and closer to the poles). This is incorrect. Statement 2: Increased tidal ranges in these areas are created by the positions and configurations of the continents in the northern hemisphere. In the higher latitudes of the northern hemisphere, the continents of North America, Europe, and Asia are pressed closer together. This “constriction” of the oceans creates the effect of a higher range of tides. In the higher latitudes of the southern hemisphere, in the southern tips of South America, southern Africa, Australia, and Antarctica, tidal ranges are not increased. In these areas the continents are not pressed closely together, there is not a “constriction” of the oceans, and the tidal ranges are not increased. Q Source: Additional Research: Page 122: Fundamentals of Physical Geography: 11th NCERT 84 Atal Ranking of Institutions on Innovation Achievements (ARIIA) will primarily focus on which of the following parameters for ranking? 1. Budget Expenses and Revenue generated through Innovation and entrepreneurship development 70

Mock Test 10 https://telegram.me/UPSCMaterials

https://telegram.me/FreeUPSCMaterials

https://telegram.me/MaterialforExam

2. Facilitating access to advance centres / facilities and entrepreneurial support system 3. International networking and partnerships fostered to promote entrepreneurship Select the correct answer using the codes below. A. 1 only B. 2 and 3 only C. 1 and 3 only D. 1 and 3 only

Correct Answer : D Answer Justification : Background: For India to emerge as a global innovation hub, the youth of our country, especially in higher education institutions (HEIs) need to play a crucial role to create a sustainable innovation ecosystem. Hence, ideally all HEIs should have a comprehensive and functional mechanism to convert research into innovations. This ecosystem will encourage, inspire and nurture young students by exposing them to new ideas and processes resulting in innovative activities in their formative years. To ensure that Innovation is primary fulcrum of all HEIs, Ministry of Human Resource Development (MHRD), Govt. of India is introducing ‘Atal Ranking of Institutions on Innovation Achievements (ARIIA)’ to systematically rank education institutions and universities primarily on innovation related indicators. ARIIA considers all major indicators which are commonly used globally to rank most innovative education institutions/ universities in the world. Justification: ARIIA will primarily focus on 5 main parameters Indicators Budget Expenses and Revenue generated through Innovation and entrepreneurship development Facilitating access to advance centres / facilities and entrepreneurial support system Idea to Entrepreneurship Development of Innovation Ecosystems Supported through Teaching & Learning Best innovative solutions Developed In-house for Improving Governance of Your Institution Total

Weightage 20

10 54 10 6 100

ARIIA ranking will certainly inspire Indian institutions to reorient their mind-set and build ecosystems to encourage high quality research, innovation and entrepreneurship. More than quantity, ARIIA will focus on quality of innovations and will try to measure the real impact created 71

Mock Test 10 https://telegram.me/UPSCMaterials

https://telegram.me/FreeUPSCMaterials

https://telegram.me/MaterialforExam

by these innovations nationally and internationally. Moreover, ARIIA will set tone and direction for institutions for future development for making them globally competitive and in forefront of innovation. Q Source: http://pib.nic.in/newsite/PrintRelease.aspx?relid=183177 85 Consider the following features of a river course. 1. Smaller tributaries flowing over gentle gradients 2. Streams meander freely forming ox-bow lakes 3. Sharper turns are not found frequently. The above refer to which stage of a river? A. Youth stage B. Old stage C. Mature Stage D. Either Youth stage or Mature Stage

Correct Answer : B Answer Justification : Learning: The characteristics of each of the stages of landscapes developing in running water regimes may be summarised as follows: Youth stage Few streams that with poor integration flowing through original slopes Shallow V-shaped valleys; deep gradients Stream divides are broad with marshes, swamps etc. Waterfalls and rapids may exist where hard rocks are there Mature stage Plenty streams with good integration; mild gradients Deep V-shaped valleys Broader meandering floodplains 72

Mock Test 10 https://telegram.me/UPSCMaterials

https://telegram.me/FreeUPSCMaterials

https://telegram.me/MaterialforExam

Water divides turn sharper Waterfalls and rapids disappear Old Stage Smaller tributaries flowing over gentle gradients Divides are broad and flat with lakes Streams meander freely forming natural leeves, ox-bow lakes Most of the landscape is either at or above sea level. Q Source: Revision: Page 60: Fundamentals of Physical Geography: 11th NCERT 86 Consider the following statements about the South Asian Free Trade Area (SAFTA) agreement. 1. The SAFTA Ministerial Council (SMC) has been established comprising the Commerce Ministers of the Member States. 2. The SAFTA Agreement states that the SMC shall meet at least once every year. Which of the above is/are correct? A. 1 only B. 2 only C. Both 1 and 2 D. None

Correct Answer : C Answer Justification : Justification: Statement 1: The SAFTA Agreement was signed in 2004 during Twelfth SAARC Summit held in Islamabad. The Agreement entered into force in 2006, and the Trade Liberalization Programme commenced. The agreement established SMC. To assist the SMC, a SAFTA Committee of Experts (SCOE) has been formed. SCOE is expected to submit its report to SMC every six months. Statement 2: It should meet once every year or more often as and when considered necessary by the Contracting States. Each Contracting State shall chair the SMC for a period of one year on rotational basis in alphabetical order.

73

Mock Test 10 https://telegram.me/UPSCMaterials

https://telegram.me/FreeUPSCMaterials

https://telegram.me/MaterialforExam

This is an old article on SAFTA, the concepts are useful nonetheless. http://www.financialexpress.com/archive/what-is-safta-what-are-its-benefits/159142/ Q Source: Questions based on South Asia 87 On a World Map, a straight line joining New Delhi to Tashkent will NOT pass through which of these countries? A. B. C. D.

Pakistan China Afghanistan Uzbekistan

Correct Answer : B Answer Justification : Learning: Tashkent is the capital of Uzbekistan, so the line will definitely pass through D. It will also pass through Tajikistan which on the way. Pakistan and Afghanistan fall on the way of the line, so A and C can’t be the answer. Looking at the map below, only B can be the answer.

74

Mock Test 10 https://telegram.me/UPSCMaterials

https://telegram.me/FreeUPSCMaterials

https://telegram.me/MaterialforExam

Q Source: Map based questions: World 88 Which of these constitutional provision(s) aim at protecting the linguistic interests of minorities in the states? 1. President can direct the government to provide compulsory financial grants and reservation in public jobs to such minorities. 2. President can direct the official recognition of a minority language in the state. Which of the above is/are correct? A. 1 only B. 2 only C. Both 1 and 2 D. None

Correct Answer : B Answer Justification : Justification: Statement 1: There is no such provision for reservation in public jobs, however it shall be the endeavour of every State and of every local authority within the State to provide adequate facilities for instruction in the mother-tongue at the primary stage of education to children belonging to linguistic minority groups. The President may issue such directions to any State as he considers necessary or proper for securing the provision of such facilities. Statement 2: When the President (on a demand being made) is satisfied that a substantial proportion of the population of a state desire the use of any language spoken by them to be recognised by that state, then he may direct that such language shall also be officially recognised in that state. Q Source: Revision of Chapter 7: Indian Polity: M Laxmikanth 89 Consider the following about Heat Waves. 1. Climate change is leading to increased frequency and intensity of heat waves. 2. In India, it is most commonly experienced in North-Western regions. 3. As per NDMA guidelines, when local temperature is constantly above forty degrees Celsius, heat waves must be declared by local authorities. Select the correct answer using the codes below. A. 1 and 2 only B. 3 only C. 2 and 3 only D. 1, 2 and 3

75

Mock Test 10 https://telegram.me/UPSCMaterials

https://telegram.me/FreeUPSCMaterials

https://telegram.me/MaterialforExam

Correct Answer : A Answer Justification : Justification: Statement 1: Climate change induced global warming, and urbanization, deforestation etc have further increased instances of heat waves. Statement 2: It is a period of abnormally high temperatures, more than the normal maximum temperature that occurs during the summer season in the North-Western parts of India. Heat Waves typically occur between March and June, and in some rare cases even extend till July. Statement 3: When actual maximum temperature remains 45 degrees C or more irrespective of normal maximum temperature, heat waves should be declared. Higher daily peak temperatures and longer, more intense heat waves are becomingly increasingly frequent globally due to climate change. There are other specific NDMA guidelines as well in this regards. Q Source: Improvisation: Past year UPSC Qs on Climate Change related to heat waves 90 Intellectual Property Rights (IPR) includes A. B. C. D.

Patents and trademarks only Trademarks, patents, geographical indications and industrial designs Innovation copyrights and plant variety protection only Circuit Designs, copyrights and industrial designs only

Correct Answer : B Answer Justification : Learning: The term Intellectual Property (IP) reflects the idea that its subject matter is product of mind or intellect. These could be in the form of patents; trademarks; geographical indications; industrial designs; semiconductor integrated circuits layout-design; plant variety protection and copyright. IP, protected through law, like any other form of property, can be a matter of trade, i.e., it can be owned, bequeathed, sold or bought. All aspects of IPRs are administered by Department of Industrial Policy and Promotion (DIPP). These are administered through the Office of the Controller General of Patents, Designs and Trade Marks (CGPDTM), Q Source: Improvisation: Past year UPSC Qs on IPR 91 With reference to National Student Startup Policy (NSSP), consider the following statements. 76

Mock Test 10 https://telegram.me/UPSCMaterials

https://telegram.me/FreeUPSCMaterials

https://telegram.me/MaterialforExam

1. The AICTE has been drafted this policy to increase the number of “Student Startups” and prepare them to gain the benefits from “Start-up India” programme. 2. It aims to create 100,000 students owned technology start-ups and a million new employment opportunities by 2025 or within the next 10 years. 3. According to the policy, the curriculum pattern followed by academic institutions registered with AICTE would include 40 per cent skills based courses, 30 per cent knowledge related courses and 30 per cent attitude related courses. Select the correct answer using the codes below. A. 1 only B. 1 and 2 only C. 1, 2 and 3 D. 2 and 3 only orrect Answer : C

Answer Justification : Justification: The formulated policy has outlined the role of the AICTE, TBI and academic institutions in creating student entrepreneurs all along implementing the government’s initiative the “Start-up India”. According to the policy, the curriculum pattern followed by these institutions would include 40 per cent skills based courses, 30 per cent knowledge related courses and 30 per cent attitude related courses. The new policy has made a provision to set up a fund to support start-up events and fest that would be organized at national and international levels. An amount of Rs. 10,000 crore will be invested through the venture funds registered with SEBI and interested to support student startups. To develop hard and soft infrastructure like testing labs, IT labs, tools room, design studios, data set, laboratories, video-conferencing facilities and research and analysis labs in the academic institutes, an initial annual outflow of Rs. 20 crore will be provided to help student start-ups in institutes. The policy will encourage science and technology students to choose entrepreneurship as their career and motivate them to convert their research projects into a workable startup. It will work to indoctrinate social responsive attitude among AICTE approved institutes’ students who would aspiring to launch social business start-ups. The policy aims to provide hand-holding support to students from rural areas that need training to start a business and identifying business opportunity near to them. Equip rural and urban students with the right set of skills and resources for launching, managing and running their business enterprises successfully so that they can contribute to the socioeconomic progress of India.

77

Mock Test 10 https://telegram.me/UPSCMaterials

https://telegram.me/FreeUPSCMaterials

https://telegram.me/MaterialforExam

Q Source: http://pib.nic.in/newsite/PrintRelease.aspx?relid=183177 92 Non-performing Assets (NPAs) are loans made by a bank or finance company on which repayments or interest payments are not being made on time. How do high NPAs affect the citizens (customers) in India? 1. They may lead to an increase in the cost of capital for borrowers. 2. They may lead to bank defaults and thus lower people’s confidence in the banking system affecting their savings ratio. Which of the above is/are correct? A. 1 only B. 2 only C. Both 1 and 2 D. None

Correct Answer : C Answer Justification : Justification: Statement 1: It results in inflating the cost of capital for economic activities and banks may charge higher interest rates on some products to compensate NPAs. NPA is any asset of a bank which is not producing any income. It affects the profitability & liquidity of the banks. Statement 2: It adversely affects the value of bank in terms of market credit and widens assets and liability mismatch. This may even lead to bank defaults. It shakes consumer’s confidence in the banking deposits, and forces them to withdraw savings in banks, thus lowering the savings ratio. Learning: Savings ratio is the total amount of a person’s income not spent on consumption and invested in either bank deposits, bonds or other market instruments. In any economy, savings finance investments. Higher savings increase liquidity in the market and reduce the cost of credit resulting in higher economic growth. So, it is very important that the banking system of an economy is robust. Q Source: Frequently in news 93 The Living Planet Programme (LPP) is a programme within the A. B. C. D.

NASA European Space Agency World Wildlife Fund IUCN 78

Mock Test 10 https://telegram.me/UPSCMaterials

https://telegram.me/FreeUPSCMaterials

https://telegram.me/MaterialforExam

Correct Answer : B Answer Justification : Learning: It is managed by the Earth Observation Programmes Directorate under the ESA. It is crucial that we continue to learn more about our planet if we are to understand the Earth system and its processes, especially within the context of global change. This will equip us better for predicting the effects a changing climate may bring. It is, therefore, also important to address scientific questions that have a direct bearing on societal issues humankind will face in the coming decades. As our quest for knowledge continues to grow, so does our demand for accurate satellite data to be used for numerous practical applications for protecting and securing the environment. Responding to these needs, ESA's Living Planet Programme comprises a science and research element, which includes the Earth Explorer missions, and an Earth Watch element, which is designed to facilitate the delivery of Earth observation data for use in operational services. LPP consists of two classes of Earth observation missions (listed below) including research missions known as Earth Explorers, and the Earth Watch class of missions whose objective is to develop support operational applications such as numerical weather forecasting or resource management. ESA has been dedicated to observing Earth from space ever since the launch of its first Meteosat meteorological satellite in 1977. Following the success of this first mission, the subsequent series of Meteosat satellites, ERS-1, ERS-2 and Envisat provided us with a wealth of valuable data about Earth, its climate and changing environment. Q Source: http://www.esa.int/Our_Activities/Observing_the_Earth/The_Living_Planet_Programme/ESA_s_Living _Planet_Programme 94 Ecosystem is formed, most appropriately, by the interaction of A. B. C. D.

Both biotic and abiotic factors of the environment All Living organisms present in the system Diverse Landforms found within a large zone Communities found near edge of a biome

Correct Answer : A Answer Justification : Justification: An ecosystem is a system formed by the interaction of all living organisms (biotic) with each other and with the physical and chemical factors (abiotic) of the environment in which they live, all linked by transfer of energy and material. 79

Mock Test 10 https://telegram.me/UPSCMaterials

https://telegram.me/FreeUPSCMaterials

https://telegram.me/MaterialforExam

Biome is a larger region than ecosystem, and comprises many such ecosystems. So, D is incorrect. We will be covering more questions on the hierarchies of ecology. Q Source: Page 5: Geography NCERT 7th: Our Environment 95 Some of the major works of Begum Rokeya Sakhawat Hossain include 1. Sultana’s Dream 2. Abarodhbasini 3. Padmarag Select the correct answer using the codes below. A. 1 and 2 only B. 2 and 3 only C. 1 and 3 only D. 1, 2 and 3 only orrect Answer : D

Answer Justification : Justification: Commonly known as Begum Rokeya, she was a Bengali writer, thinker, educationist, social activist, advocate of women's rights. She wrote novels, poems, short stories, science fiction, satires, treatises, and essays. In her writings, she advocated that both men and women should be treated equally as rational beings, and the lack of education is the main reason of women's lagging behind. Her major works include Abarodhbasini, a spirited attack on the extreme forms of purdah that endangered women's lives and thoughts; Sultana's Dream, a science fiction novella set in a place called Ladyland in nisah, a world ruled by women; Padmarag ("Essence of the Lotus", 1924), another feminist utopian novel; Matichur, collection of essays in two volumes. Rokeya suggested that education of women is the foremost requisite of women's liberation; hence she established the first school aimed primarily at Bengali Muslim girls in Kolkata. th

Q Source: Chapter 5: NCERT 7 : Social and Political Life 96 Poles experience about six months day and six months night due to which of the following? 1. Axial Tilt of earth 2. Coriolis Force 3. Revolution of Earth around the Sun Select the correct answer using the codes below. A. 1 and 2 only B. 2 and 3 only 80

Mock Test 10 https://telegram.me/UPSCMaterials

https://telegram.me/FreeUPSCMaterials

https://telegram.me/MaterialforExam

C. 1 and 3 only D. 1 only

Correct Answer : C Answer Justification : Justification: Statement 1: Earth is tilted in such a way that even after one complete rotation North Pole will remain dark as it is tilted away from the Sun. See image below. So, A must be an answer. Statement 3: If the earth did not revolve around the Sun, but only rotated on its axis, we would see one Pole of the earth permanently facing day or permanently facing night. It would never change. You can visualize this. So, revolution of the earth around the Sun changes the direction of the tilt that faces the Sun, and hence brings day/night to the other pole. So, C must also be an answer. Statement 2 is absurd.

Q Source: Revision questions: Previous Test Syllabus 97 Consider the following statements. 1. The literacy rate of general population as per Census 2011 is 73% but for STs it is less than 50%. 2. Most of the North-eastern states perform worse than the other Indian states in terms of tribal literacy. Which of the above is/are correct? 81

Mock Test 10 https://telegram.me/UPSCMaterials

A. B. C. D.

https://telegram.me/FreeUPSCMaterials

https://telegram.me/MaterialforExam

1 only 2 only Both 1 and 2 None

Correct Answer : D Answer Justification : Justification: Statement 1: Despite government efforts to promote education among the Scheduled Tribes (STs), their literacy rates as compared to the national average have remained low, a Parliamentary Committee has said this week. The literacy rate as per Census 2011 is 73% but for STs is 59% only. Statement 2: The overall literacy gap amongst the various groups and STs has come down from 19.77% in 1961 to 14.03% in 2011, a scrutiny of state-wise literacy data reveals that in most of the north eastern states like Meghalaya, Mizoram and Nagaland, STs are at par with the general population.

Q Source: Additional Research in tribal educational covered in Chapter 5: NCERT 7th: Social and Political Life 98 Optically-Stimulated Luminescence is a dating technique that is used to find out A. B. C. D.

The last time quartz or feldspar sediment was exposed to light Radiation emitted by antecedent water bodies Age of tree barks by exposing them to ionizing radiation Duration of photons incident on any surface

Correct Answer : A Answer Justification : Learning: Indian researchers have used dating techniques on animal remains and pottery fragments to conclude that the Indus Valley settlements could be older than previously believed. They also used the technique called Optically-Stimulated Luminescence. It is a late Quaternary dating technique used to date the last time quartz sediment was exposed to light. As sediment is transported by wind, water, or ice, it is exposed to sunlight and zeroed of any previous luminescence signal.

82

Mock Test 10 https://telegram.me/UPSCMaterials

https://telegram.me/FreeUPSCMaterials

https://telegram.me/MaterialforExam

Once this sediment is deposited and subsequently buried, it is removed from light and is exposed to low levels of natural radiation in the surrounding sediment. The technique measures their age using ionizing radiation. You can read more here http://timesofindia.indiatimes.com/india/Indus-era-8000-years-old-not-5500-ended-because-of-weake r-monsoon/articleshow/52485332.cms Q Source: Surprise questions 99 Direct Heat of the Sun can be felt immensely at Ladakh because of A. B. C. D.

High water vapour saturation in air Day temperatures are very high in Ladakh Thin air at Ladakh High albedo of ice as compared to regions down the hill

Correct Answer : C Answer Justification : Justification: The altitude in Ladakh varies from about 3000m (in Kargil) to more than 8,000m (in the Karakoram). Due to such a high altitude, the climate is extremely cold and dry. So, option A and B are wrong. Option D is also wrong because albedo of Ice reflects the sunlight, whereas here we are talking about direct rays of the Sun. The air at this altitude is so thin that the heat of the sun can be felt intensely. The area experiences freezing winds and burning hot sunlight. You will be surprised to know that if you sit in the sun will your feet in the shade, you may suffer from both sunstroke and frost bite at the same time. Learning: The day temperatures in summer are just above zero degree and the night temperatures well below 30°C. It is freezing cold in the winters when the temperatures may remain below 40°C for most of the time. As it lies in the rain shadow of the Himalayas, there is little rainfall, as low as 83

Mock Test 10 https://telegram.me/UPSCMaterials

https://telegram.me/FreeUPSCMaterials

https://telegram.me/MaterialforExam

10 cm every year. Q Source: Page 74: Geography NCERT 7th: Our Environment 100 Consider the following statements. 1. At low river speeds, downward cutting dominates bank cutting. 2. At lowest reaches of the river, erosion tendencies lead to formation of many distributaries. Which of the above is/are correct? A. 1 only B. 2 only C. Both 1 and 2 D. None

Correct Answer : D Answer Justification : Justification: Statement 1: At higher gradients, downward, vertical erosion is more dominant. This produces V-shaped valleys. As gradients decrease, downward erosion is less dominant, and lateral erosion begins to dominate, forming meanders. Statement 2: Floodplains are created as a result of both erosion and deposition. When the river flows normally its bed is raised through accumulation of deposits. Material is also deposited on the sides forming raised banks called Levees. When the river reaches the sea, the fine materials it has not dropped yet are deposited at its mouth, forming a fan shaped alluvial area called a delta. Q Source: Revision of Chapter 5: Goh Cheng Leong - Certificate Physical and Human Geography

84

Related Documents

Ias 10
May 2020 35
Prelims
November 2019 54
Prelims
November 2019 58
Ias
April 2020 39

More Documents from ""

Dns Economy Bits
August 2019 64
Indian_polity.pdf
August 2019 39
Mhcet-2017 (1).pdf
November 2019 34
Tarn Aka
August 2019 32